Stat Pearls - Drugs, Pharmacology, Mechanism of Action (Cardiac, Respiratory, Analgesic, Opiod Overdose, Med Math, Antibiotics, Anti-diabetic, etc.)

Ace your homework & exams now with Quizwiz!

A 65-year-old man with poorly controlled diabetes mellitus type 2 presents with complaints of gastric fullness and constipation. The clinician adds metoclopramide to his current medications. The client asks the nurse how this drug works. Which of the following would be the most appropriate response? Well done! You answered successfully 1. "It reduces acid production in the stomach." 2. "It decreases motility of the stomach." 3. ''It promotes stomach emptying." 4. ''It decreases blood glucose levels.''

''It promotes stomach emptying." This client has developed diabetic gastroparesis. Metoclopramide is FDA-approved for the treatment of diabetic gastroparesis. It increases gastric motility and promotes gastric emptying. Metoclopramide blocks the antiperistaltic effects of apomorphine.

What is the correct single dose of oral acetaminophen in a child weighing 12 kg? A. 325 mg B. 120 mg C. 250 mg D. 650 mg

120 mg The correct oral acetaminophen dose is 10 to 15 mg/kg per dose. The correct range of dosing for a 12 kg patient should be between 120 and 180 mg. Acetaminophen poses a risk for severe hepatic toxicity after single ingestion of minimum toxic doses. In healthy children aged 1 to 6 years, this is 150 to 200 mg/kg. In adults, it is 7.5 to 10 g. Acetaminophen overdose can cause end-organ toxicity which manifests 24 to 48 hours after an acute ingestion and initial asymptomatic phase.

A 65-year-old woman presents to the clinician for opioid dependency. The clinician prescribed medication to treat opioid-dependency in a patient who had a contraindication to methadone. The medication is a partial agonist at the mu receptor. What is the half-life of this medication after sublingual administration? A. Less than 2 hours B. Between 4-6 hours C. 10-14 hours D. 25-70 hours

25-70 hours When administered orally, buprenorphine has poor bioavailability because of the first-pass effect. The majority of the drug is broken down by the liver and intestine. Sublingual administration is the preferred route of administration. The absorption is fast, and this route also avoids the first-pass effect. Once the tablet is placed under the tongue, it has a slow onset of action, with the peak effect occurring at 3-4 hours after administration. Once in the body, buprenorphine is broken down by the cytochrome CYP 34A enzymes to an active metabolite, norbuprenorphine, with weak intrinsic activity. The average half-life of buprenorphine is about 38 hours, with a range of 25-70 hours, following sublingual administration.

A patient with alcohol intoxication presents to the psychiatry department and has been prescribed chlordiazepoxide due to worsening symptoms. What is the half-life of this drug? 1. 10 to 30 hours 2. 10 to 50 hours 3. 20 to 70 hours 4. 30 to 100 hours

30 to 100 hours The half-life of chlordiazepoxide is 30 to 100 hours. The dose of chlordiazepoxide can be tapered up in case of worsening alcohol withdrawal symptoms. The dose of chlordiazepoxide is tapered down to lessen the sedation. Thiamine supplements are an integral part of therapy for the treatment of alcohol withdrawal and intoxication.

A 68-year-old female with colon cancer is being prepped to undergo a hemicolectomy with general anesthesia and an epidural for postoperative pain management. She did a bowel prep the day prior and only had clear liquids, and has been NPO since 8 pm. Her weight is 50 kg. She has an 18 G peripheral IV cannula in place. Her incision will be medium-sized, from her belly button to her pubic symphysis. Her case starts at 8 am. Once she has been put under general anesthesia, how much crystalloid would she need per hour to account for insensible fluid losses? A. 300 mL/hour starting in the first hour of surgery B. 300 mL/hour starting in the second hour of surgery C. 400 mL/hour starting in the first hour of surgery D. 400 mL/hour starting in the second hour of surgery

300 mL/hour starting in the second hour of surgery Insensible fluid losses are calculated based on the size of the incision. With a medium-sized incision, the average insensible losses are 4-7mL/kg/hr. This patient's insensible fluid losses will be around 6mL/kg/hr. 50 kg x 6 mL/kg/hr = 300 mL/hour. Most anesthesiologists do not include insensible fluid losses in the first hour, given that most of the first hour is used for induction of general anesthesia, line placement, positioning of the patient, prepping, and draping. The surgeon's incision likely did not occur until in the first hour, so it is most reasonable to start to replace the fluid deficit of the patient and give maintenance IV fluids, and not include insensible fluid loss in the first hour of surgery. Insensible fluid losses of 400 mL/hour would be correct for a large incision, which is usually dosed at 8-10 mL/kg/hour depending on the size of the incision. 8 mL/kg x 50 kg = 400 mL/hour for this particular patient.

A healthy 75-year-old woman presents to the clinic for a follow up of her hypertension. According to the patient, her blood pressure has been well controlled for the past three decades with a diuretic, and she has no complaints. However, she has been smoking one pack per day for the past four decades. Upon further inquiry, the patient explains that she has no allergies and that she has never had surgery. Which of the following preventive interventions is most appropriate for this patient? A. A drug that irreversibly inhibits both COX 1 and 2 B. A drug that is a non-selective beta receptor antagonist C. A drug that is a prostanoid selective FP receptor agonist D. A drug that inhibits HMG-CoA reductase

A drug that irreversibly inhibits both COX 1 and 2 The United States Preventive Services Task Force has supported the use of daily aspirin as a preventive intervention for patients with at least two risk factors for cardiac disease. These risk factors include male sex, premature coronary artery disease in the first-degree relative, smoking, hypertension, diabetes mellitus, a history of stroke, peripheral vascular disease, obesity, and low levels of high-density lipoprotein cholesterol. Aspirin is a cyclooxygenase-1 (COX-1) inhibitor. It is a modifier of the enzymatic activity of cyclooxygenase-2 (COX-2). Unlike other NSAIDs (ibuprofen/naproxen), which bind reversibly to this enzyme, aspirin binding is irreversible. It also blocks thromboxane A2 on platelets in an irreversible fashion preventing platelet aggregation. The most common side effect of aspirin is gastrointestinal upset ranging from gastritis to gastrointestinal bleed.

A 34-year-old woman presents for advice on contraceptive options. She states that she delivered a baby one month ago via spontaneous vaginal delivery, and she has been using condoms since the delivery. Her past medical history is significant for pulmonary embolism and DVT four years ago, chronic liver disease, migraine without aura, coronary artery disease, and polycystic ovarian syndrome (PCOS). She admits to occasional tobacco smoking. She denies drinking alcohol or using illicit drugs. Her family history is negative for breast and gynecologic cancer. Physical examination reveals a blood pressure of 112/78 mmHg, a pulse of 78/min, a respiratory rate of 12 breaths/min, and a temperature of 37 C (98.6 F). Which of the following patient factors is an absolute contraindication to the use of oral contraceptive pills in this patient? A. Liver disease B. History of venous thromboembolism C. Age plus smoking D. Coronary artery disease

A history of pulmonary embolism, stroke, ischemic heart disease, and venous thromboembolism are absolute contraindications to using oral contraceptive pills (OCPs). Uncontrolled hypertension and migraine with aura are also absolute contraindications for the use of OCPs. OCPs are contraindicated in those who smoke and are over 35 years of age due to the significant risk of cardiovascular events and deep vein thromboembolism. This patient is under 35 years of age, and she is an occasional smoker. Relative contraindications include coronary artery disease, migraine without aura, hypertension, liver disease, and diabetes mellitus. One of the non-contraceptive benefits of OCPs is treating polycystic ovarian syndrome (PCOS).

Which "starch blocker" medication can decrease postprandial hyperglycemia? A. Glyburide B. Metformin C. Acarbose D. Pioglitazone

Acarbose inhibits degradation and absorption of complex carbohydrates from the intestine. Postprandial hyperglycemia is decreased by acarbose because it decreases glucose absorption after meals by slowing the digestion of complex carbohydrates. Acarbose works if taken at the beginning of a meal. The recommendation is to take it with the first bite of each meal, usually 3 times a day. The medication is usually started low and titrated up slowly to decrease gastrointestinal side effects.

A farmer presents with excessive salivation, sweating, rhinorrhea, and lacrimation. What neurotransmitter is responsible for this presentation? Well done! You answered successfully A. Epinephrine B. Acetylcholine C. Dopamine D. Serotonin

Acetylcholine Cholinergic toxicity is caused by excess amounts of acetylcholine at the synapse of muscarinic and nicotinic receptors. In this case, the patient is experiencing cholinergic toxicity due to organophosphate poisoning. Excess acetylcholine results from either enhancement of acetylcholine that is already present at the synapse or the prevention of acetylcholine from being broken down at the synapse; this is done by mimicking acetylcholine or blocking acetylcholinesterase from working. Acetylcholinesterase is the enzyme that degrades acetylcholine at the synapse to yield its byproducts. Cholinergic toxicity occurs when there is too much acetylcholine available in the receptor synapse, causing an excess level of parasympathetic effects. The other neurotransmitters listed do not work at muscarinic or nicotinic receptors and do not cause an excess of acetylcholine; therefore, they can not result in cholinergic toxicity.

A 23-year-old female patient presented to the emergency department after her roommate found her lying on their living room floor unconscious. Her blood pressure is 100/70 mmHg, pulse 60/min, respirations 6/min, and temperature 98.0 F (36.7 C). The patient is unable to cooperate with a physical examination. Her pupils are contracted bilaterally. Her airway is secured. A workup shows the patient took an overdose of a medication that increases the duration of chloride channel opening. What is the most appropriate next step in the management of this patient? A. Flumazenil B. Alkalinize the urine C. Activated charcoal D. Naloxone

Activated Charcoal Barbiturates are a class of sedative-hypnotic drugs. They are commonly used as antiepileptics (phenobarbital) and for the induction of general anesthesia (thiopental). Some states administer barbiturates for provider-assisted suicide/euthanasia and use them for capital punishment by lethal injection. Barbituates potentiate GABA-A receptors and increase the duration of chloride channel opening. Even though barbiturates mediate their effects via GABA, flumazenil is not effective in the treatment of barbiturate toxicity. Barbiturates have no antidotes, and the treatment is supportive. Activated charcoal is beneficial in the management of a barbiturate overdose. However, alkalizing the urine has not been shown to be effective. Treatment of barbiturate toxicity remains supportive as there is no specific antidote or overdose. The first step in treatment, as with any overdose, is assessing the patient's airway, breathing, and circulation. With significant sedation and respiratory depression, intubation and mechanical ventilation may become necessary. Early treatment with activated charcoal may be useful and can be given via a nasogastric tube.

A 55-year-old man presents to the healthcare provider after he develops a red rash with itching. He had recently been started on vancomycin for an infection. What is the most appropriate step in the management of this patient? A. Decrease the vancomycin infusion rate B. Administer antihistamines C. Administer steroids D. Administer epinephrine

Administer antihistaminesThe vancomycin flushing syndrome is most commonly seen with vancomycin therapy, although other antibiotics such as amphotericin B, ciprofloxacin, teicoplanin, and rifampicin can cause it. A few rare cases have been reported with cefepime, but it is most closely associated with vancomycin.The syndrome appears within minutes after rapid intravenous administration and presents with a red pruritic rash over the face and trunk and the sensation of diffuse burning and itching. The rash is due to mast cell degranulation and the release of histamine. Less frequently, hypotension and angioedema can occur.In more severe cases, patients may become agitated and develop headaches, chills, fever, chest pain, and shortness of breath.The vancomycin flushing syndrome can be treated by administering an antihistamine and discontinuing the infusion. In mild cases, once the rash and itching subside, the infusion can be resumed at a slower rate.

A 43-year-old female patient with a past medical history of hypertension and diarrhea-predominant irritable bowel syndrome presents with complaints of persistent abdominal pain and diarrhea ongoing for the past few weeks. She has had tried multiple over-the-counter medications without significant relief. Normally she is able to control her symptoms with antispasmodics, increasing her dietary fiber intake and stress management, but it has not worked this time. She reports feeling increasing amounts of stress due to her job. She denies eating anything unusual, taking antibiotics recently, or travel outside the country. She does not report any fevers, chills, weight loss, hematochezia, or melena. She is afebrile, blood pressure is 128/72 mmHg, the pulse rate is 82/min, respiratory rate is 18/min, and she is maintaining saturation at 98% on room air. On physical examination, she is well-appearing, cardiovascular, and respiratory exams are within normal limits. The abdomen is soft, non-tender, no distension and bowel sounds are present. What is the next best step for the management of this patient's condition? A. Start the patient on antibiotics B. Advise alternative strategies for stress management C. Perform a colonoscopy D. Start the patient on therapy with alosetron

Advise alternative strategies for stress management To meet the criteria to be started on alosetron, women with severe diarrhea-predominant irritable bowel syndrome (IBS-D) had to have chronic symptoms lasting six months or longer, must not have any anatomic or biochemical abnormalities of the GI tract placing them at risk for complications of constipation (CoC) and instances of ischemic colitis (IC), and had to have failed conventional therapy. Since the patient is normally able to control her symptoms with conventional therapy, she reports increased stress due to her job, so the best option is to advise alternative strategies to stress management. Because she has responded to conventional therapy in the past and her current exacerbation has been more than 6 months, she does not qualify for alosetron. She does not appear to have infectious diarrhea that could warrant the use of antibiotics in such a patient.

A 65-year-old man presents to the emergency department with a right leg weakness that started two hours ago. Vital signs are a pulse of 82/min, blood pressure of 125/68 mmHg, respiratory rate of 18/min, and temperature of 98.6 F (37 C). On examination, the power in the right leg is reduced compared to the left leg. CT head reveals ischemic stroke. He is given a drug that forms a product that relieves the obstruction. Which of the naturally occurring enzymes inhibit the action of this formed product? 1. Alpha 2-antiglobulin 2. Alpha 2-antitrypsin 3. Enteropeptidase 4. Alpha 2-antiplasmin

Alpha 2-antiplasmin Tissue plasminogen activator (tPA) is classified as a serine protease (enzymes that cleave peptide bonds in proteins). Its primary function includes catalyzing the conversion of plasminogen to plasmin, the primary enzyme involved in dissolving blood clots.The alpha 2-antiplasmin is a serine protease inhibitor (serpin) and inhibitor of plasmin, an enzyme that participates in fibrinolysis and the degradation of various other proteins.The alpha 2-antiplasmin is encoded by the SERPINF2 gene. This enzyme quickly inactivates plasmin and restricts the action of plasmin to the vicinity of the clot.In liver cirrhosis, there is decreased production of alpha 2-antiplasmin, leading to decreased inactivation of plasmin and an increase in fibrinolysis.

A group of clinical researchers is investigating the management of osteoarthritis in patients with underlying psychiatric disorders. Patients recruited for the study are analyzed based on their medication regimen. 25% of patients in the study take a medication that binds to opioid receptors. What is a primary indication for the use of the drug mentioned above? A. Anxiety B. Analgesia C. Insomnia D. Inflammation

Analgesia Tramadol is an opioid pain medication, and like other opioids, selectively bind to different opiate receptors in the central nervous system. The liver enzyme, CYP2D6, converts tramadol to its active metabolite M1, which has a stronger affinity for the mu receptor compared to the inactive form. Tramadol also inhibits serotonin and norepinephrine reuptake. Indications include relief of moderate to severe pain. It can be a contributing factor to serotonin syndrome, so care should be exercised to note other medications in a patient regimen that also can increase serotonin levels.

A 92-year-old female patient is admitted to the nursing home after a short hospital stay for a fractured right hip. She has been taking clindamycin for an infected right heel pressure ulcer and has now developed watery diarrhea. Her blood pressure is 83/52 mmHg and the heart rate is 115/min. Her family is concerned that she has not been eating and drinking well. What is the most likely cause of this patient's current condition? A. Poor intake B. Sepsis C. Antibiotics D. Poor nursing home care

Antibiotics Clindamycin is notorious for causing clostridium dificile infection of the colon with copious diarrhea. While the intake of this patient may have decreased recently, her main problem is increased fluid losses from diarrhea. Given that the patient is dehydrated, a fluid bolus and IV maintenance fluids should be administered first, and the patient re-assessed before jumping to a diagnosis of sepsis and septic shock. Postoperative delirium is common in the elderly and could be occurring in this case, but was not described in the stem and is not the best answer choice. Most nursing homes expect that patients are able to feed themselves, if this patient is unable to do so, she may need to be transferred to a higher level of care.

An 82-year-old man is admitted to the hospital for dyspnea and sputum production. He has a long-standing history of chronic obstructive pulmonary disease (COPD) and has been admitted three times over the past six months. He currently uses ipratropium, prednisone, roflumilast, and 4 L/min home oxygen. He is diagnosed with another episode of COPD and given appropriate treatment. As part of his management, palliative options are discussed and he states he would like to pursue palliative care moving forward. Which of the following therapies is most appropriate to be offered as part of his future treatment? A. Intubation B. Lobectomy C. Morphine D. Montelukast

Appropriate palliative therapies for COPD include bronchodilating inhalers, home oxygen, and morphine.Although morphine suppresses the respiratory drive, it is used to decrease the anxiety associated with dyspnea in patients with COPD.Intubation and lobectomy are aggressive treatments, not appropriate for palliative care. Montelukast is not recommended in patients with COPD.The patient has chosen palliative care as his future COPD management. Palliative care for COPD includes offering therapies that improve the patient's symptoms or anxiety, without the aim of improving mortality (possibly worsening mortality).

A 17-year-old woman with a past medical history significant for asthma presents with shortness of breath, difficulty breathing, and wheezing. It started about an hour after she woke up and has continued to worsen. Oxygen saturation is 87%. She is tachypneic, exhibits accessory muscle usage, and has diffuse expiratory wheezing on auscultation. It was determined that a drug class that exhibits phosphodiesterase inhibition could have helped to prevent the onset of her current acute symptoms. Which of the following is a side effect of this drug? A. Miosis B. Anuria C. Arrhythmia D. Central nervous system depression

Arrhythmia The patient is most likely suffering from an asthma exacerbation. Factors making this likely are the patient's prior history, shortness of breath, hypoxia, and diffuse expiratory wheezing. The drug class, which acts to prevent acute asthma exacerbations and exhibits phosphodiesterase inhibition, is methylxanthines. Methylxanthines, such as theophylline, exhibit side effects such as cardiac arrhythmias, central nervous system stimulation, tremor, diuresis, nausea, vomiting, and headaches. Miosis would be expected in a drug exhibiting cholinergic action. Anuria is the opposite of the diuresis that can occur with the administration of methylxanthines. CNS stimulation can occur with the administration of methylxanthines.

A 65-year-old male presents with the complaint of continuous bleeding after a dental extraction, which was done five days ago. His past medical history is significant for coronary artery disease, hypertension, and arthritis. He denies any prescription anticoagulants and stopped taking celecoxib three days before the procedure. His vitals are stable, and his exam reveals a small oozing area from the extraction site without signs of infection or arterial bleeding. What is the most likely medication affecting his clotting factors? A. Diclofenac B. Ibuprofen C. Acetaminophen D. Aspirin

Aspirin This patient has a history of coronary artery disease. Aspirin is frequently prescribed to this patient population and typically not listed by patients because it is over the counter. Given the patient is already on a prescription COX-2 inhibitor, it is unlikely he would be on another NSAID as well. Aspirin binds covalently and irreversibly to cyclooxygenase (COX) enzymes, permanently affecting the lifespan of platelets, which is usually one week. The binding of aspirin to COX results in a conformational change of the enzymes and inhibits the metabolism of arachidonic acid. NSAIDs other than aspirin bind reversibly to COX and therefore decrease the synthesis of prostanoids from arachidonic acid. Thus, stopping a COX-1 or COX-2 inhibitor within a few days of a procedure will eliminate any bleeding issue. Acetaminophen is a centrally acting nonopioid analgesic that has weak action on COX-2.

A patient presents with episodes of palpitations, dizziness, tremors, and feeling faint. She says these symptoms generally occur if she does not eat breakfast. Which of the following is most appropriate for the evaluation of this patient? A. Check for diabetes B. Dipstick urine for ketones C. Assess symptoms and diet D. Check blood sugars

Assess symptoms and diet It appears that this patient has hypoglycemic symptoms. Hypoglycemia typically presents with palpitations, tremors, anxiety, hunger, and feeling faint. Others with hypoglycemia may present with confusion, blurred vision, and even severe lethargy. Her symptoms have to be evaluated in relation to her diet. She may have hypoglycemia or post-prandial hyperglycemia.

A 22-year-old man presents to the emergency department in pain after being injured during a college baseball game. He states that he twisted his ankle while running, and fears that he has an ankle sprain. His history is significant for asthma and two episodes of depression over the past year. He admits to smoking half a pack of cigarettes daily and smoking marijuana over the weekends with his friends. On examination, his vitals include a blood pressure of 124/83 mmHg, a pulse of 89/minute, respiration of 13/minute, and a temperature of 98 F (36.6 C). On physical examination, he is unable to bear weight on his right ankle. While waiting for a radiology and orthopedic consult, he requests pain medication. Which of the following in the patient's history is the strongest contraindication to giving him oxycodone? A. Smoking cigarettes B. Smoking marijuana C. Asthma D. Depression

Asthma Oxycodone belongs to a group of drugs known as opioid analgesics. It works in the brain to change how an individuals body feels and responds to pain. Respiratory side effects of oxycodone seen are respiratory depression and cough. Oxycodone therapy, therefore, is contraindicated in patients with asthma. It is also contraindicated in patients with respiratory depression, hypercarbia, hypersensitivity to oxycodone, and known or suspected ileus or gastrointestinal obstruction, Alternate acute pain medications should be used in asthmatic patients requiring acute relief of pain.

A 55-year old man presents to the hospital for a routine visit. The patient's medical history is significant for hypertension. His blood pressure is 150/100 mm Hg and his pulse rate is 82/min. He is started on low doses of a medication that blocks the cardiac myocyte adrenergic receptors and reduces the heart rate and contractility. This drug should be used cautiously in which of the following conditions? A. Asthma B. Peptic ulcer disease C. Congestive heart failure D. Myocardial infarction

AsthmaBeta receptors exist in three distinct forms: beta-1 (B1), beta-2 (B2), and beta-3 (B3). Beta-1 receptors located primarily in the heart mediate cardiac activity.Catecholamines, epinephrine, and norepinephrine bind to B1 receptors and increase cardiac automaticity as well as conduction velocity. B1 receptors also induce renin release, and this leads to an increase in blood pressure.Beta-blockers work by blocking beta-adrenergic stimulation resulting in a decrease in cardiac output, myocardial contractility, myocardial oxygen demand, blood pressure, and heart rate.Beta-blockers should be used with caution in asthmatics since they induce bronchospasm in such patients.

An ophthalmologist is performing a cataract repair on a 67-year-old female. Prior to the procedure when gaining consent from the patient, the clinician places a few drops of atropine-based solution in the patient's eye being operated on to keep the pupil dilated during the procedure. What is the mechanism of action of atropine in this case? A. Stimulates alpha receptors of the pupillary dilator muscle. B. Inhibits muscarinic receptors of the pupillary constrictor muscle. C. Inhibits nicotinic receptors in the Edinger-Westphal nucleus (CN III). D. Inhibits acetylcholinesterase enzymes in the synaptic cleft of the pupillary dilator muscle.

Atropine is classified as an anticholinergic, inhibiting all three types of muscarinic receptors in the body decreasing parasympathetic neurotransmission. Atropine is used topically in ophthalmic surgery in order to keep the pupils dilated for easier access to the optic disc and retina. The topical solution achieves only local delivery of atropine, which has a higher affinity for muscarinic receptors than nicotinic receptors. This route of administration takes advantage of atropine's anticholinergic effects on the eye without having to account for its other systemic effects of increasing blood pressure and heart rate. Atropine has other uses besides pupillary dilation, including the correction of severe bradycardia. Atropine is contraindicated in those with glaucoma especially angle-closure, and in children due to its hyperthermic effects.

A carpenter is placed on neuroleptic medications while hospitalized and shows extrapyramidal system side effects. Which of the following would be the most important educational point for after discharge? A. Avoid sun exposure due to photosensitivity B. Drink plenty of fluids but avoid caffeine and alcohol C. Avoid using sharp instruments and power tools D. Stand slowly to avoid dizziness

Avoid using sharp instruments and power tools Extrapyramidal system symptoms can include tremors, sudden muscle spasms, and muscular rigidity. These could make using sharp instruments and power tools dangerous. If a patient is experiencing acute onset of EPS, particularly dystonia, the provider must assess if an emergency airway intervention is necessary as laryngeal and pharyngeal dystonic reactions may increase the risk of imminent respiratory arrest. Dystonic reactions are rarely life-threatening, and the provider should discontinue the offending agent and manage pain if present.

A 17-year-old male presents to the primary care office with a cough that has a rust color sputum as well as feeling fatigued and a fever. The symptoms have been occurring for three days and states he needs a doctor note due to missing his employment. On physical examination, his vital signs are a blood pressure of 125/87 mmHg, heart rate of 84/min, respiratory rate of 22/min, and a temperature of 102.3ºF. On physical examination, there is persistent tactile fremitus and decreased breath sounds in the left lower lung field. The physician orders a chest X-Ray that shows a consolidation in the left lower lung field. What is the most appropriate antibiotic for the management of this patient? A. Azithromycin B. Cefazolin C. Clindamycin D. Linezolid

Azithromycin The patient is most likely presenting with an acute case of community-acquired pneumonia (CAP). Some of the first-line agents for CAP are macrolides or tetracyclines. Azithromycin also has coverage for atypical bacteria causing pneumonia. One of the criteria used in determining hospitalizations for Pneumonia is the CURB-65.

A 43-year-old patient with a history of diabetes, asthma, obesity, and hyperthyroidism comes to the clinic for a routine checkup. The patient has a family history of hypertension in both the parentsand his older sister. He was recently diagnosed with hypertension and started on propranolol by the healthcare provider two weeks age. On further questioning, the patient reports having to use hisrescue inhaler more often over the last week. Which of the following should be said to the patient? A. "Your metered-dose inhaler should be changed." B. "You should stay indoors to avoid asthma exacerbations. "C. "Metformin can interact with beta-blockers and cause dyspnea." D. "Beta-blockers can lead to constriction of small airways."

Beta-blockers can lead to constriction of small airways.This patient is using his asthma inhaler more frequently soon after he was started on a non-selective beta-blocker, propranolol, two weeks ago.Beta-blockers are frequently used in the treatment of hypertension. These drugs act to decrease blood pressure in multiple ways. They bind to beta-adrenergic receptors on the sinoatrial node of the heart and decreases it's firing rate. This results in lower heart rate which leads to decreased cardiac output. Beta-blockers also act on the kidneys to decrease the secretion of renin, which results in arteriolar vasodilation, thus decreasing the blood pressure. It is possible for non-selective beta-blockers like propranolol to bind to beta receptors on the smooth muscle cells of the small airways and cause bronchcoconstriction.

A 22-year-old female suffers from seasonal allergies as well as mild persistent asthma. She is currently on fluticasone/salmeterol and salbutamol. The physician prescribes a new asthmatic prophylaxis asthmatic medication to control both her comorbidities. Which of the following best describes the mechanism of action for this medication? 1. Non-selective histamine blocker 2. Blocks the action of leukotriene D4 3. Binds with allergens, preventing them from activating mast cells 4. Stabilizes immunologically sensitized mast cells

Blocks the action of leukotriene D4 Montelukast comes in tablet, chewable tablet, and granular forms. Montelukast can be utilized to treat both seasonal allergies and be prophylactic asthma medication. Montelukast (empirical formula C35H35ClNNaO3S) is a highly selective leukotriene receptor antagonist that binds with a high affinity to the cysteinyl leukotriene receptor for leukotrienes D4 and E4. These leukotrienes are excreted by various types of cells, such as mast cells, and are involved in the inflammatory process that may cause the signs and symptoms of asthma and allergic rhinitis. Leukotriene receptors are found in airway cells, such as macrophages and smooth muscle cells. When bound to leukotriene receptors, montelukast inhibits leukotriene physiologic effects (such as airway edema, smooth muscle contraction, and impairment of normal cellular activity) without exhibiting any agonist activity. In asthmatics, low doses of montelukast (5 mg) induce a significant inhibition of bronchoconstriction caused by leukotriene D4. Because of this mechanism of action, it is only used prophylactically. It does not serve as a 'rescue' medication.

A 65-year-old male presents to the emergency department with complaints of severe chest pain, profuse sweating, and vomiting for the past 1 hour. An electrocardiogram (ECG) tracing is suggestive of ventricular tachycardia. He is given a medication that reverts the patient to a normal sinus rhythm. Twenty minutes after administration of the drug, the patient develops slurred speech, tinnitus, circumoral numbness, and tingling. Discontinuation of the medication is necessary to prevent progression to what adverse effects? A. Disseminated intravascular coagulation (DIC) B. Pulmonary embolus C. Cardiovascular collapse D. Cerebrovascular accident (CVA)

Cardiovascular collapse This patient with signs of slurred speech, tinnitus, and circumoral numbness/tingling after taking a medication to treat his ventricular tachyarrhythmia likely has lidocaine toxicity. The patient should be examined for possible central nervous system (CNS) toxicity, including restlessness, agitation, and paranoia, among others. These signs can signify impending cardiorespiratory collapse if actions are not taken immediately. Additional monitoring parameters for systemically administered lidocaine include electrocardiograms and vital signs. Patients can become hypotensive and tachycardic, and arrhythmias can result. The patient should be kept in a supine position to minimize the hypotensive effects of the drug. Patients with a suspected local anesthetic overdose should be treated immediately with intravenous lipid emulsion.

A 53-year-old male with a history of hyperlipidemia and hypertension presents with a new-onset headache. He has no other complaints. His vitals show a blood pressure of 210/185 mmHg, a heart rate of 60bpm, a respiratory rate of 15/min and oxygen saturation of 100% on room air. On examination, there is a bilaterally distributed headache without tenderness, and no signs of acute trauma. The patient is given a medication to decrease his blood pressure. After some time, the patient begins to feel palpitations in his chest and his heart rate increases to 120 bpm. What is the most likely cause of this change? A. Allergic reaction B. Reflex sympathetic discharge C. Release of norepinephrine D. Inhibition of cholinergic activity

Cardiovascular side effects from nitroglycerine use include hypotension, bradycardia, and reflex tachycardia. Central nervous system side effects include headaches, dizziness, and flushing. It can be taken by mouth, sublingually, or transdermally. It also can be administered intravenously. Nitroglycerine is indicated for angina pectoris, acute myocardial infarction chest pain, and pulmonary edema with hypertension. An allergic reaction causes breathing difficulties, urticaria, and low blood pressure. Nitroglycerine does not have cholinergic activity. It acts on the smooth muscles of the vessels.

A 12-year old girl develops sudden shortness of breath, cough, and wheezing after being injected with an antibiotic in the hospital. On examination, her blood pressure is 80/60mm Hg, and her pulse is 120/min. Her face is flushed and swollen, and there are diffuse wheezes throughout her chest. Further inquiry reveals that she had been injected with an antibiotic that is a cell wall synthesis inhibitor. The drug acting on which of the following receptors is the most appropriate to administer in an emergency? A. Catecholamine receptors B. Dopamine receptors C. Histamine receptors D. Cholinergic receptors

Catecholamine receptors The first thing to do with any drug rash is to stop the offending drug and avoid the use of that drug in the future. Anaphylaxis is characterized by wheezing, facial swelling, and hypotension. This requires urgent treatment with adrenaline and emergency management of blood pressure and airway. A penicillin allergy that causes hives or a rash is best treated with an antihistamine, but severe reactions may require epinephrine. Patients with an allergic reaction to drugs may be observed overnight. If a severe reaction occurs, an overnight observation is recommended.

A 17-year-old female presents to the clinic for painful urination and pus-like vaginal discharge. Social history shows she has multiple sex partners, and she does not use protection when having intercourse. The culture of the discharge shows gram-negative diplococci. Which of the following cephalosporin is most likely to be used to treat the infection? A. Ceftriaxone B. Cefepime C. Ceftaroline D. Cephalexin

Ceftriaxone Third-generation cephalosporin ceftriaxone is commonly used to treat gonorrhea. Third-generation cephalosporins have much greater gram-negative activity than previous generations as well as increased coverage against Enterobacteriaceae, Neisseria, and H. influenza. Third-generation cephalosporins have less activity against most gram-positive organisms. Ceftriaxone is only administered parenterally.

A 17-year-old male presents to the clinic for a small rash one week after returning from a family hiking trip in Connecticut. The patient states that the rash seems to be getting bigger. The examination of the rash shows a red rash with a clear center. Which of the cephalosporins can be used to treat the suspected infection? A. Cefuroxime B. Cephalexin C. Ceftazidime D. Cefepime

Cefuroxime The patient is displaying signs and symptoms of early Lyme disease, a disease caused by Borrelia burgdorferi infection. Second-generation cefuroxime has coverage against Lyme disease in pregnant women and children. Cefuroxime can be administered orally or parenterally. Cefuroxime is also approved for Haemophilus influenza meningitis infection.

A 48-year-old woman visits a clinic with back pain. The provider plans to start the patient on a nonsteroidal anti-inflammatory drug. She has a history of sulfa allergy. Which of the following nonsteroidal anti-inflammatory drugs cannot be given to this patient? A. Aspirin B. Ibuprofen C. Celecoxib D. Piroxicam

Celecoxib is a type of sulphonamide agent. Allergy is the body's immune response to foreign substances common in the environment and triggers a reaction from the body's immune response described as hypersensitivity. The most common triggers of allergy/anaphylaxis include foods, medications, insect stings, and allergen immunotherapies. Celecoxib should not be used or used very carefully in patients with a reported sulfa allergy in the past.

A 35-year-old male is prepared for abdominal surgery but is noted to have low blood pressure. Which of the following would be the mechanism of action for an agent used for the induction of general anaesthesia by the anaesthetist that would address this issue? A. Centrally mediated sympathetic effects B. Negative inotropic effect C. Analgesic effect D. Parasympathetic effects

Centrally mediated sympathetic effects Ketamine is often used in patients with shock or low blood pressure as it has a centrally mediated sympathetic effect that can cause an increase in heart rate, cardiac output and blood pressure. Ketamine has a negative inotropic effect that may be unmasked if a patient is catecholamine depleted. Ketamine is also a potent analgesia but does have psychotropic properties. Unwanted mental side effects of ketamine can be avoided by using low doses of benzodiazepines.

A 17-year-old female is brought to the emergency department by her mother. She looks anxious and withdrawn. Her mother says she ingested 18 tablets of acetaminophen three hours ago. The girl appears oriented to time, place, and person. She currently has no symptoms. Physical examination is unremarkable. Her vitals include BP 110/60 mmHg, pulse 98 per minute, fever 99 degrees celsius, and respiratory rate of 19 per minute. She is admitted to the hospital. Intravenous access is placed. Blood tests are ordered. Her lab reports are as follows: Hb 12.3 g/dL, TLC 6.8 X 10^9/L, Platelets 201,000/mm3, BUN 13 mg/dL, serum creatinine 0.8 mg/dL, Na 135 mEq/L, K 3.3 m3q/L, Cl 99 mEq/L, PT 12 seconds, APTT 35 seconds, ALT 34 IU/L, AST 29 IU/L. Which of the following is the most likely cause of morbidity and mortality in this patient? A. Renal failure B. Sepsis C. Cerebral edema D. Myocardial Infarction

Cerebral edema The patient ingested a large amount of acetaminophen in an attempted suicide. She appears stable right now (might be until 24 hours), but she has to be admitted to the hospital for observation if she progresses to acute liver failure from acetaminophen poisoning. Acetaminophen is toxic to the liver if ingested more than 4gm in 24 hours. Cerebral edema is the most likely cause of mortality in these patients. Exhaustion of hepatic metabolic pathways causes the increased formation of a toxic metabolite of acetaminophen. Hyperammonemia is neurotoxic and impairs numerous biochemical functions. The patient might have non-specific symptoms initially but may show nausea, RUQ pain, and lethargy. Labs show a deranged coagulative profile and abnormal LFTs. Management includes monitoring acetaminophen levels four hours after ingestion. N-acetylcysteine might be administered as an antidote per oral. Liver transplantation might be needed as a last resort. Kidney failure occurs in approximately 50% of patients but is an uncommon cause of mortality. Sepsis can lead to acute liver failure, not vice versa. MI is not associated with morbidity and mortality.

A 65-year-old woman is admitted to the intensive care unit with severe urosepsis. Her heart rate is 120/min, respiratory rate 26/min, blood pressure 90/50 mmHg, and temperature 101 F (38.3 C). Examination demonstrates suprapubic tenderness. Urine and blood cultures both grow Proteus mirabilis. She is started on meropenem but does not improve. Molecular testing reveals the isolate carries an inosine monophosphate (IMP) carbapenemase mutation. What is the most appropriate antibiotic regimen for this patient while awaiting formal sensitivities? A. Add ciprofloxacin to the meropenem B. Change the meropenem to gentamicin C. Add colistin to the meropenem D. Change the meropenem to tigecycline

Change the meropenem to gentamicin Because this patient has not shown improvement on meropenem and the isolate has shown carbapenemase activity, the antibiotic regimen must be changed. Proteus mirabilis has intrinsic resistance to polymixins and tetracyclines. Piperacillin-tazobactam and gentamicin should be considered. Susceptibility testing should guide therapy. Gentamicin is the best second-line treatment, and rifampin can be used at third-line treatment. Combining two or more antibiotics has been linked to improved outcomes in carbapenem-resistant infections.

A 32-year-old man with a past medical history of acid reflux, hyperlipidemia, and hypertension is found to have an uncomplicated skin structure infection due to methicillin-susceptible Staphylococcus aureus. He is started on ofloxacin 400 mg orally every 12 hours for 10 days. In addition, he takes calcium carbonate 500 mg orally 6 times daily, lisinopril 10 mg orally daily, and atorvastatin 20 mg orally nightly. On day 8, it appears his infection is worsening. What is the most likely reason for the worsening infection? A. Incorrect diagnosis B. Incorrect antibiotic choice C. Ofloxacin-lisinopril interaction D. Chelation of ofloxacin

Chelation of ofloxacin The patient is taking an antacid. Fluoroquinolones chelate di- and tri-valent metal cations, thus antacids or vitamins containing calcium, magnesium, aluminum, iron, and zinc should be avoided during ofloxacin therapy, as these reduce the amount of drug that gets absorbed. In cases of severe infection, intravenous administration is preferred as it allows higher dosing that leads to higher drug concentrations, ultimately leading to an increase in clinical cure rates. Ofloxacin can be used to treat uncomplicated skin and skin structure infections due to methicillin-susceptible Staphylococcus aureus, Streptococcus pyogenes, or Proteus mirabilis. There is no reason to believe the patient has methicillin-resistant Staphylococcus aureus as there is an obvious reason given for the antibiotic to not be working.

A 65-year-old male patient presents with a history of cough, sore throat, and fever for the past four days. On examination, his throat looks inflamed, and there is purulent exudate visible on the peritonsillar areas. He is started on a bacteriostatic antibiotic. Which of the following is the most likely drug? A. Amphotericin B B. Oxacillin C. Cefepime D. Clarithromycin

Clarithromycin Clarithromycin is part of the macrolides class of antibiotics. Clarithromycin works by inhibiting the 50S ribosomal subunit, thus halting protein synthesis. When protein synthesis is halted, the microorganism cannot grow and thus halts all functions. Bacteriostatic antibiotics are often paired with bacteriocidal antibiotics to take advantage of synergistic effects in eradicating infection. Macrolides have a wide range of uses ranging from pneumonia, skin infections, as well as sexually transmitted infections.

An adult male is diagnosed with an anxiety disorder and started on benzodiazepines. Which of the following ions is involved in mediating the action of benzodiazepine? A. Sodium B. Chloride C. Potassium D. Magnesium

Chloride Benzodiazepines produce their effects by modification of the gamma-aminobutyric acid (GABA) receptor. This causes an increase in the conductance of the chloride channel. When benzodiazepines bind to GABA, there is increased conduction of chloride across the neuronal cell membrane. Benzodiazepines also act as adenosine reuptake inhibitors which may account for their muscle relaxant effects. Benzodiazepine overdose is treated with flumazenil. Benzodiazepines are used to treat anxiety, alcohol dependence, seizures, pain attacks, and agitation.

A 12-year-old boy presents with dyspnea, chest pain, and a cough productive of thick green sputum. He has a past medical history of cystic fibrosis. His vital sign shows oxygen saturation 98% on room air, respiratory rate 24 per minute, heart rate 105 beats per minute, blood pressure 125/80 mmHg, and temperature 40.2 C (104.4 F). On examination, wheezes, rhonchi, and crackles are heard. There is dullness to the percussion of the chest. Sputum culture shows Pseudomonas aeruginosa. Cefepime is prescribed. Which of the following is the most appropriate adjuvant antibiotic? A. Ciprofloxacin B. Trimethoprim-sulfamethoxazole C. Doxycycline D. Amoxicillin

Ciprofloxacin Ciprofloxacin can be used in combination with other antibiotics to cover P. aeruginosa. Previous studies on juvenile animals have displayed cartilage damage and arthropathy due to quinolone use. Due to this, quinolone use in pediatric patients is limited. Some quinolones are approved by the food and drug administration (FDA) for pediatric use in cases such as inhalation anthrax, complicated urinary tract infections, pyelonephritis, plague, conjunctivitis, otitis, sinusitis, respiratory tract infections, pneumonia, and gastrointestinal diseases. Common side effects in pediatric patients include gastrointestinal events (nausea, vomiting, diarrhea, clostridium difficile-associated colitis) and arthralgias. Excessive use of quinolones (mostly in the adult population) has led to the increased emergence of bacterial resistance.

A 65-year-old man is on numerous medications following an acute myocardial infarction. He is in the intensive care unit for close monitoring. Now, he is lethargic and has slurred speech. Which of the following medications is the most likely cause of the patient's symptoms? Well done! You answered successfully A. Nitrate B. Nonsteroidal anti-inflammatory drug C. Class Ib antiarrhythmic medication D. HMG-CoA reductase inhibitor

Class Ib antiarrhythmic medication Lidocaine hydrochloride is an antiarrhythmic drug capable of toxic or adverse reactions on the CNS. Early CNS symptoms include perioral numbness and tongue paresthesia. More serious central nervous system symptoms include lethargy, confusion, drowsiness, and slurred speech. Large lidocaine overdoses can result in unconsciousness leading to coma.

A cystic fibrosis patient is admitted to the ICU with a complicated right lower lobe pneumonia on 4L oxygen by nasal cannula. Initial sputum stain demonstrated Gram-negative rods, and the patient was given cefepime. He continues to have high fevers despite antibiotics. The lab calls and reports that they suspect a carbapenem-resistant enterobacteriaceae is the pathogen. What is the backbone of double or triple combination therapy? A. Colistin B. Tigecycline C. Aminoglycoside D. Carbapenem

Colistin is considered by most to be the backbone for most carbapenem-resistant enterobacteriaceae (CRE) treatment. Tigecycline is the backbone for colistin-resistant CRE. Aminoglycosides may play a part in double or triple therapy but are not considered integral to improving outcomes. Carbapenems may play a part in double or triple therapy but are not considered integral to improving outcomes.

A 66-year-old patient is brought to the emergency department following the intentional ingestion of an unidentified drug in a suicide attempt. His past medical problems include hyperlipidemia and chronic back pain. His pupils are miotic and he has a respiratory rate of 12 per minute. Which is the possible drug that he might have ingested? A. Atorvastatin B. Atropine C. Codeine D. Cocaine

Codeine Miosis and respiratory depression are signs of opioid toxicity. Codeine and other opioids are not only involved in intentional but also unintentional overdose in chronic users . Treatment of toxicity depends on the symptoms and degree of intoxication and involves symptomatic therapy like enema, respiratory intubation, and definitive therapy with an opioid antagonist. The drug of choice to reverse toxication is naloxone, which is an opioid antagonist.

A 62-year-old female presents to the emergency department after a fall while bathing. She states that she fell on the wet floor and is in severe pain. Her history is significant for hypertension, osteoporosis, diabetes, and a gastric ulcer. Physical examination reveals an inability of the patient to bear weight on her right hip. Radiological consult confirms a hip fracture. After appropriate management, she is discharged on Oxycodone as needed for pain. Which of the following side-effect is most common with this analgesic? A. Headache B. Burning urination C. Epigatrium pain D. Constipation

Constipation Oxycodone belongs to a group of opioid drugs known as opioid and is used to help relieve moderate to severe pain. The side effect profile of oxycodone is similar to that of the other opioid medications. Constipation is the most common overall side effect. Other reported side effects of oxycodone include nausea, vomiting, lightheadedness, bradycardia, hypotension, palpitations, dizziness, or drowsiness. Headache may occur but is less common. To prevent constipation, it is recommended to eat dietary fiber, exercise, and drink enough water.

A 25-year-old man presents with pain (8/10) because of a motor vehicle accident. The provider decides to treat the pain with an opioid analgesic. Which of the following is a long-term common adverse effect of opioid analgesic treatment which may develop? A. Central nervous system stimulation B. Constipation C. Insomnia D. Nausea

Constipation is a common long-term side effect of opioid analgesics. Other common side effects include pruritis, nausea, vomiting, and headache. Most resolve over time. Opioid analgesics usually have a sedative effect, but insomnia can occur. The main concern with opioid analgesic use is their abuse potential and overdose-related fatalities.

A 59-year-old male presents to the clinic with a six-month history of severe low back pain, which began after a motor vehicle collision. Radiographs were taken immediately after the accident were negative for fractures or acute pathology. The patient has treated his pain with NSAIDs and acetaminophen with no relief. He has attempted physical therapy for six weeks, with no relief of his pain. The patient's blood pressure is 130/82 mm Hg, pulse 60/min, respirations 16/min, and temperature 98.0 F (36.7 C). The clinician prescribes a low dose of hydrocodone-acetaminophen for use on an as-needed basis. Which of the following side effects is not likely to lessen with continued use of the medication? A. Nausea B. Constipation C. Euphoria D. Sedation

Constipation is a side effect of opioid analgesics that do not lessen with continued use and tolerance. Often a high fiber diet or laxatives should be considered. Opioids act both presynaptically and postsynaptically to produce an analgesic effect. Adverse effects include dysphoria, euphoria, sedation, respiratory depression, constipation, suppression of endocrine systems, cardiovascular disorders (e.g., bradycardia), convulsion, nausea, vomiting, pruritus, and miosis. Nausea is more likely to decrease with continued use and tolerance. Euphoria is a side effect of opioid analgesia that is more likely to decrease with continued use and tolerance at the same dosage. Similarly, sedation is a side effect of opioid analgesia, which lessens with continued use and tolerance at the same dosage. Opioids can cause fatal overdose through respiratory depression, especially when combined with other sedatives such as alcohol and benzodiazepines. Patients with altered mental status, depressed respiration, and constricted pupils should be suspected of suffering from an acute opioid-related overdose, which can be fatal if untreated. Overdose can be reversed by agents such as naloxone, which can be given intravenously, intramuscularly, or intranasally. Naloxone is a centrally-acting pure opioid antagonist with a high affinity, which quickly counteracts opioid action. Naloxone can be given in small, repeated doses and titrated to a desirable response.

A 65-year-old female patient with a history of type 2 diabetes mellitus and chronic atrial fibrillation comes into the office with a prescription for glyburide. Upon reviewing her medication profile, it becomes evident that she is on concurrent therapy with warfarin and sulfadiazine. Which of the following is the most appropriate next step? A. Ask the patient about her history with hypoglycemic symptoms B. Advise the physician to ensure that the dosage of glyburide is the lowest available C. Continue to fill the prescription as written D. Consult the physician about possible drug-drug interactions with her concurrent medications

Consult the physician about possible drug-drug interactions with her concurrent medications Glyburide, when taken with concurrent medications metabolized by CYP2C9, increases the risk of hypoglycemia in patients with diabetes mellitus type 2. Glibenclamide or glyburide is an antidiabetic drug in a class of medications known as sulfonylureas, which are closely related to sulfonamide antibiotics. Glyburide therapy is not completely contraindicated with other sulfonamide medications or warfarin, but therapy must be closely monitored for hypoglycemia if concurrent medications are taken together. An intervention must be done about this patient, as all three medications she has been prescribed are metabolized through CYP2C9. This poses a potentially harmful interaction and increases hypoglycemic risks.

A patient on 4 liters of oxygen by nasal cannula has an arterial blood gas performed. The patient has normal work of breathing, with a respiratory rate of 23. The arterial blood gas indicates a pH of 7.40, PaO2 = 92 mmHg, and PaCO2 = 42 mmHg. Which of the following is most appropriate nursing action? A. Decrease the oxygen to 2 liters B. Continue present management and record the data in the chart C. Increase the oxygen to 6 liters D. Encourage the patient to breathe faster

Continue present management and record the data in the chart The arterial blood gas shows the patient is receiving adequate treatment, and no changes are necessary. A normal physiologic range of PaO2 is 75-100 mmHg. If someone is requiring supplemental oxygen you need to asses WHY, instead of ONLY increasing the oxygen. There are multiple reasons for hypoxemia: V/Q mismatch, right-to-left shunt, diffusion impairment, hypoventilation, and low Fraction of inspired O2. at sea level, low inspired O2 is not the cause for hypoxemia. You can evaluate quickly an intubated patient with hypoxemia using the pneumonic DOPE. Displacement (moved endotracheal tube), Obstruction (secretions or object to be suctioning or removed from the airway), Pneumothorax, Equipment failure (detached ventilator, wrong ventilator settings, malfunctioning ventilator, malfunctioning surveillance equipment).

A 28-year-old male presents to the ED with signs of alcohol withdrawal. The patient states he hasn't had a drink in 18 hours but drinks heavily on a daily basis. He has no pertinent medical history or surgical history. As you place orders for his care, what should be a key part of your plan for this patient? A. Strict bed rest B. Continuous monitoring of vitals C. Nonsteroidal anti-inflammatory drugs D. Vitamins and high protein diet tray

Continuous monitoring of vitals Alcohol withdrawal can occur as early as 6 hours and up to 72 hours after a patient's last drink. Abnormal vital signs indicate the progression of the withdrawal due to the autonomic nervous system becoming unstable. This includes tachycardia, hypertension, and tachypnea. Delirium tremens is the most severe form of alcohol withdrawal and can result in significantly altered mental status and autonomic hyperactivity. Alcohol withdrawal seizures should be treated similar to all causes of seizures, initially with benzodiazepines.

A 26-year-old patient presents with pain in his neck. He states the pain is worse in the morning and improves throughout the day. He states that it localizes to the upper region of his neck, and he states that he has also noted some eye swelling. His most recent fasting blood sugar was 250mg/dl. What is the next preferred management? A. Oral sulindac, intravenous hydrocortisone B. Control blood sugar, oral sulindac C. Oral methotrexate, control blood sugar D. Intravenous hydrocortisone, oral methotrexate

Control blood sugar, oral sulindac Ankylosing spondyloarthritis presents in young men with neck stiffness that improves throughout the day and can be associated with anterior uveitis.There are many treatments that can be employed in their treatment, Administration of NSAID agents including sulindac is one of them.There are two contraindications of sulindac: a recent history of coronary artery bypass grafting and a history of hypersensitivity reactions to NSAIDs.Administration of corticosteroid agents without control blood sugar may worsen the situation.

Acetaminophen interferes with the excess release of prostaglandins by inhibiting which enzyme? A. Lipoxygenase B. Cyclooxygenase C. Phospholipase C D. Phospholipase A2

Cyclooxygenase NSAIDS and acetaminophen inhibit cyclooxygenase. Cyclooxygenase is responsible for producing prostaglandins and thromboxanes. Lipooxygenase is involved with producing leukotrienes, not prostaglandins. Phospholipase inhibition may inhibit the production of prostaglandins but are not inhibited by acetaminophen.

A 24-year-old man presents with abdominal pain 24 hours after undergoing an appendicectomy. He describes the pain as eight out of ten. He is given a 30 mg single dose of ketorolac, and his abdominal pain goes down to 2/10. What is the mechanism of action of this drug? A. Cyclooxygenase inhibitor B. Myeloperoxidase inhibitor C. Inhibits primarily cyclooxygenase 2 (COX-2) D. Directly inhibits the production of thromboxane A-2

Cyclooxygenase inhibitor Ketorolac is a food and drug administration (FDA) approved medication used in the treatment of moderate to severe acute onset pain. It is a nonsteroidal anti-inflammatory drug (NSAID) class. Ketorolac works by inhibiting cyclooxygenase enzyme, which converts arachidonic acid into prostaglandins, prostacyclin, and thromboxane. The inhibition of these molecules decreases pain. Ketorolac does not inhibit myeloperoxidase. It does not specifically inhibit cyclooxygenase-2. It inhibits both cyclooxygenase 1 and 2. It does not directly inhibit thromboxane A-2.

A 17-year-old male patient displays symptoms of attention deficit hyperactivity disorder (ADHD). The provider seeks to prescribe a non-amphetamine-based drug. What is the mechanism of this drug based on the most likely medication being prescribed? A. Dopamine transporter (DAT) and norepinephrine transporter (NET) blocker B. Direct alpha-agonist activity C. Dopamine transporter (DAT) blocker D. Selective serotonin reuptake inhibitor

Dopamine transporter (DAT) and norepinephrine transporter (NET) blocker Methylphenidate is the most likely medication to be prescribed in this clinical scenario. Methylphenidate works to block the reuptake of dopamine and norepinephrine. Methylphenidate is an agent used to maintain wakefulness and treat attention deficit hyperactivity disorder. It blocks dopamine and norepinephrine reuptake by neurons. Methylphenidate is a well-tolerated stimulant drug that is not an amphetamine.

A patient presents to his primary care provider with blurry vision and trouble swallowing that is worse at the end of the day. He is diagnosed with Myasthenia Gravis and is started on treatment. What sympathetic nervous system mediated effect may Myasthenia Gravis result in? A. Tachycardia B. Bronchodilation C. Decreased sweating D. Constipation

Decreased sweating Myasthenia gravis is an autoimmune disease characterized by anti-acetylcholine receptor antibodies which block the action of acetylcholine at synaptic junctions and the neuromuscular junction. The only sympathetic nervous system response mediated with acetylcholine listed among the answer choices is sweating. Blocking acetylcholine will result in decreased sweating. Besides the action of acetylcholine in sweating in the sympathetic nervous system, acetylcholine is the major transmitter of the parasympathetic system and neuromuscular junction. Tachycardia, constipation, and bronchodilation are sympathetic nervous system responses, but they are mediated by the neurotransmitter norepinephrine. Blocking the parasympathetic nervous system neurotransmitter acetylcholine may result in tachycardia, bronchodilation, and constipation, sympathetic nervous system responses, but this is achieved by blocking the parasympathetic nervous system and not the sympathetic nervous system.

A 34-year-old patient with ankylosing spondylitis presents for a follow-up visit. His only complaint is that of epigastric pain that improves with eating. Upon further evaluation, he is diagnosed with a duodenal ulcer that is helicobacter pylori-negative. How could sulindac result in this condition? A. Decreased perfusion to efferent arteriole B. Decreased synthesis of gastroprotective prostaglandins C. Decreased synthesis of systemic prostaglandins D. Decreased synthesis of thromboxane A2

Decreased synthesis of gastroprotective prostaglandins Sulindac is a non-selective NSAID agent.Non-selective NSAID agents work by inhibiting the synthesis of prostaglandins and thromboxane A2.Decreased synthesis of gastroprotective prostaglandins results in peptic ulcer disease.Decreased synthesis of thromboxane A2 results in the antiplatelet effects of NSAIDs.

A 55-year-old female presents to the clinic for routine care. The client has a history of hypertension and diabetes for the last 20 years. She works as a landscaper for a local company and has been managing chronic stable angina with nitroglycerin spray. Which of the following best describes the mechanism of the therapeutic effect seen in this client? 1. Decreased myocardial contractility 2. Decreased blood pressure 3. Decreased venous constriction 4. Decreased arterial constriction

Decreased venous constriction Stable angina is defined as substernal chest pain that increases with activity and decreases with rest.Nitroglycerin is a vasodilator, which has a greater effect on the veins.Dilation of large veins leads to a decrease in cardiac venous return, due to which the heart has to pump less blood. This decrease in cardiac workload leads to a decreased myocardial oxygen demand, which alleviates angina.Nitroglycerin does not decrease the myocardial contractility. Neither a decreased arteriolar constriction nor decreased blood pressure is the major mechanism of angina palliation by nitroglycerin.

Which sedative drug has spasmolytic activity? A. Propofol B. Donepezil C. Diazepam D. Succinylcholine

Diazepam is effective as an antispasmodic agent and an antiseizure drug. It is used short term for anxiety as there are issues with tolerance and dependence. Diazepam is used to treat acute alcohol withdrawals, epilepsy, and muscle spasms. Diazepam binds to the GABAA receptor which helps the inhibitory neurotransmitter gamma-aminobutyric acid (GABA) open this chloride channel.

A 59-year-old woman presents for preoperative evaluation of her left knee for total knee replacement. She has a history of progressively-worsening osteoarthritis in her left knee for 10 years, which has been treated with physical therapy, naproxen, and occasional intra-articular methylprednisolone acetate. However, now her activity is markedly restricted due to severe pain on movement. Her medical history is also significant for obesity, and type 2 diabetes mellitus, for which she takes glyburide. Her family history is notable for diabetes mellitus in both her parents. Her temperature is 36.7 C (98 F), blood pressure is 137/88 mm/Hg, the pulse is 82/min, and respirations are 14/min. Her BMI is 36 kg/m2. She consumes an average of 2-3 alcoholic drinks before bed each night and has a remote smoking history of 10-pack-years. After a detailed physical examination and physical and radiological evaluation, the decision is made to proceed with the surgery at the next possible availability, for which informed consent is taken. Which of the following actions is the most appropriate for this patient? A. Continue with glyburide during the course of the surgery B. Discontinue glyburide on the morning of the day on which surgery is planned C. Double the dose of glyburide prior to the surgery for better control against neurohumoral stress response of surgery D. Only take half the dose of glyburide on the day of the surgery due to the fear of hypoglycemia

Discontinue glyburide on the morning of the day on which surgery is planned The neuroendocrine stress response associated with surgery and general anesthesia results in the release of counter-regulatory hormones, which include glucagon, cortisol, epinephrine, and growth hormone. These cause insulin resistance, impaired insulin secretion, decreased peripheral glucose utilization, increased lipolysis, which may lead to hyperglycemia (or even ketosis). Patients with type 2 diabetes mellitus, taking oral hypoglycemic drugs or non-insulin injectables, are advised to continue their medications until the morning of the surgery, at which point they should be withheld. Glyburide, glipizide, and glimepiride are second-generation sulfonylureas used in clinical practice. They are classified as insulin secretagogues. These, in particular, increase the risk of hypoglycemia due to increased insulin secretion. In the post-operative course, these oral hypoglycemic agents can be restarted as soon as the patient is able to eat well. However, a gradual step-up approach may be adopted in case of sulfonylureas, starting at a lower dose and gradually titrating it upwards.

A 46-year-old man presents after being found unresponsive by his wife. She reports going to the store, returning a few hours later, and finding him "passed out" on the couch. He has a past medical history of chronic pain and has been taking buprenorphine for opioid use disorder treatment. Examination reveals pinpoint pupils, hypoxia, cyanosis, and respiratory depression. He fails to respond to naloxone. His urine drug screen is only positive for opioids. What is the most appropriate next step in treatment?A. Administer flumazenilB. Administer doxapramC. Administer activated charcoalD. Administer theophylline

Doxapram is an intravenous drug known to stimulate respiration, increasing the respiratory rate and tidal volume.Doxapram stimulates chemoreceptors in the carotid vessels, which then stimulate the respiratory centers in the brain.Doxapram can be used to stimulate respiration in patients with opioid overdose. Be aware that it can induce panic attacks, hypertension, and excessive sweating.Flumazenil is used in the setting of benzodiazepine overdose. Activated charcoal is used for certain ingestions that occur within an hour or so of ingestion and should only be used in someone who is awake with normal mental status to reduce the risk of aspiration.

A 40-year old male comes to the physician with complaints of involuntary movements of his neck and facial muscles. He also has abnormal movements of his tongue. These symptoms have been present from the past 2 months. His past medical history is significant for schizophrenia, for which he has be taking antipsychotics. The physician notices repeated protruding movements of his tongue. The patient's symptoms are caused by medications that inhibit which of the following receptors? A. Catecholamine receptors B. Dopamine receptors C. Cholinergic receptors D. GABA receptors

Dopamine Tardive dyskinesia is syndrome that includes a group of iatrogenic movement disorders caused due to a blockade of dopamine receptors. Tardive dyskinesia is term-7caused due to long-term exposure to first and second-generation neuroleptics, certain antidpressants, lithium, and some antiemetic medications. The first-generation antipsychotics with increased dopamine D2 receptor affinity are affiliated with higher risk of casing permanent abnormal involuntary movements. A diagnosis of antipsychotic-induced tardive dyskinesia is made after the symptoms have persisted for at least one month and required exposure to neuroleptics for at least three months.

A 23-year-old man recently started taking a new medication for the long-standing history of hallucinations, poor eye contact, flat affect, and poor work performance. After six days of using this medication, this patient presents to the emergency department with acute neck stiffness. On exam, his head is rotated to the left and flexed in a fixed position. The medication used by this patient mainly works by blocking which of the following receptor? A. Muscarinic receptor B. Dopamine receptor C. Serotonin receptor D. Alpha 1 receptor

Dopamine receptor High potency dopamine receptor antagonists, such as haloperidol, are used in schizophrenia, which is characterized by hallucination, delusion, flat affect, and poor work performance. This patient has torticollis caused by haloperidol. This condition is known for acute dystonia.Dystonia occurs more commonly in young males and typically within five days of starting a new antipsychotic medication.First-line treatment for torticollis may include benztropine, an antimuscarinic medication.Other treatment modalities include benzodiazepines or other anticholinergics such as promethazine.

A 17-year-old male patient displays symptoms of attention deficit hyperactivity disorder (ADHD). The provider seeks to prescribe a non-amphetamine-based drug. What is the mechanism of this drug based on the most likely medication being prescribed? A. Dopamine transporter (DAT) and norepinephrine transporter (NET) blocker B. Direct alpha-agonist activity C. Dopamine transporter (DAT) blocker D. Selective serotonin reuptake inhibitor

Dopamine transporter (DAT) and norepinephrine transporter (NET) blocker Methylphenidate is the most likely medication to be prescribed in this clinical scenario. Methylphenidate works to block the reuptake of dopamine and norepinephrine. Methylphenidate is an agent used to maintain wakefulness and treat attention deficit hyperactivity disorder. It blocks dopamine and norepinephrine reuptake by neurons. Methylphenidate is a well-tolerated stimulant drug that is not an amphetamine.

Which of the following antidepressants is a tertiary amine and blocks the reuptake of serotonin as compared to norepinephrine? A. Nortriptyline B. Amoxapine C. Doxepin D. Maprotiline

Doxepin is a tertiary amine and blocks the reuptake of serotonin as compared to norepinephrine. Amoxapine is a tricyclic antidepressant that also has antipsychotic effects. Amoxapine is a potent inhibitor of norepinephrine reuptake. Amoxapine is the only tricyclic antidepressant that also has antipsychotic properties.

A 17-year-old patient presents to the clinic after experiencing significant dysphagia and retrosternal heartburn. He was recently prescribed medication to treat a genital chlamydial infection. Which of the following medications could have caused this patient's presenting symptoms while treating the genital chlamydial infection? A. Ceftriaxone B. Penicillin C. Doxycycline D. Alendronate

Doxycycline Tetracyclines most commonly cause GI distress in patients, such as abdominal discomfort, epigastric pain, nausea, vomiting, and anorexia. Also, while taking tetracyclines, discoloration of teeth and inhibition of bone growth in children may occur. To avoid esophagitis, it is important to consume tetracyclines with a full glass of water and remain upright following oral administration. Medication-induced esophagitis, an adverse effect of tetracyclines, may present as retrosternal heartburn and dysphagia. Ceftriaxone is used in the treatment of gonorrhea infections, which often is co-administered with tetracyclines due to co-infection with chlamydia. Alendronate can cause significant esophagitis but is not an antibiotic and not used in the treatment of chlamydial infections.

A 65-year-old female presents for increased pedal edema since the last week. She reports being on her feet for 8 hours a day at work. She has a history of hypertension, hyperlipidemia, hypothyroidism, and heart failure with preserved ejection fraction. The patient's daily medications are atorvastatin, losartan, levothyroxine, and bumetanide. On physical exam, the clinician notes pitting pedal edema bilaterally. A 2D echocardiography shows an ejection fraction of 40-45%. While educating the patient on wearing compression stockings and raising her limbs at the end of the day, the clinician adjusts her diuretic dose to twice a day. Appropriate management while monitoring this patient would include which of the following? A. Checking blood pressures two-three times a week B. Writing a work note to decrease the amount of time on her feet C. Periodically monitoring electrolytes D. Providing a referral to a nutritionist to adjust her diet

Electrolytes should be monitored periodically since there is increased excretion of sodium, potassium, calcium, and magnesium through the urine. Bumetanide is a loop diuretic that has indications to be used as first-line options for fluid overload states associated with congestive heart failure. While being less on her feet may help in the short term with the edema. The best option for edema associated with CHF would be to increase diuresis in the context of mixed heart failure. When managing a patient on a diuretic, it is essential to continue monitoring electrolytes the longer they are using it. In cases of hypokalemia and hypomagnesemia, sudden cardiac arrhythmias may develop and can lead to life-threatening situations. It is vital to check labs and replace the lost electrolytes as necessary. Replacing sodium, potassium, and calcium through oral intake may prevent the increased risk of side effects associated with depleted electrolytes.

A 58-year-old patient previously diagnosed with myasthenia gravis is seen with complaints of profound weakness and respiratory distress. On physical examination, the pulse is 52 beats per minute, the temperature is 98.9 degrees F, respiratory rate 36 breaths per minute and shallow, and blood pressure is 80/66 mm Hg. In addition, she also has bilateral wheezing, upper airway rhonchi, and also there is drooling saliva. Which of the following is the next best step in the management of this patient? A. Endotracheal intubation B. Lorazepam 2 mg IV stat C. 1 mg atropine and if there is improvement pralidoxime should be given D. If the patient has a cholinergic crisis, an atropine drip should be started

Endotracheal intubation The bradycardia, wheezing, and salivation are indicative of a cholinergic crisis. The cholinergic crisis may be difficult to differentiate from the myasthenic crisis. The management of the cholinergic crisis encompasses three stages: (1) prehospital care, (2) emergency department management, and (3) Inpatient care. Prehospital care includes the initial stabilization of the patient and the removal of the offending toxic agent. Decontamination should be initiated as soon as possible if poisoning with organophosphate or nerve gas is the primary culprit of the cholinergic crisis. All clothing should be removed from the patient's body to prevent continued contamination and to prevent the cross-contamination of first responders. Regardless of the etiology of cholinergic crises, the core principle in stabilization is ABC: airway, breathing, and circulation. Inpatient care includes continued cardiopulmonary support and monitoring. Patients with a cholinergic crisis should be admitted to the intensive care unit.

A 55-year-old man receives a chloroprocaine spinal for open inguinal hernia repair. He has a history of hypertension controlled with two medications, hypercholesterolemia, knee osteoarthritis, and obstructive sleep apnea. He is 5 feet 9 inches tall and weighs 85 kg. A 50 mg dose of chloroprocaine is injected into the intrathecal space. Surgery is uneventful, and estimated blood loss is minimal. In the post-anesthesia care unit, the patient complains of significant back pain, leg myalgias, and malaise. What spinal adjunct was likely administered to this patient? 1. Fentanyl 2. Neostigmine 3. Clonidine 4. Epinephrine

Epinephrine should not be administered with spinal chloroprocaine as it has been associated with flu-like symptoms. The pathophysiology is unclear. The combination of spinal chloroprocaine and epinephrine has been linked to the development of the following flu-like symptoms: malaise, myalgias, arthralgias, fever, nasal congestion, and loss of appetite. Fentanyl has been shown to prolong the surgical block when co-administered with chloroprocaine without adversely affecting time to discharge. Little research has been conducted regarding the co-administration of neostigmine or clonidine with chloroprocaine. Neither drug has been linked to the development of flu-like symptoms.

A 55-year-old patient with delirium that is not caused by alcohol or drug withdrawal is being treated with haloperidol. Which of the following best represents the adverse effects of this drug? A. Extrapyramidal disorder B. Hyperactivity C. Urinary retention D. Metabolic derangements

Extrapyramidal Disorder Haloperidol is an antipsychotic drug.It causes a wide variety of side effects like dryness of the mouth, excess drooling, constipation, or diarrhea.It may also cause heartburn, loss of appetite, or blurred vision.It may also cause extrapyramidal symptoms like acute dyskinesias and dystonic reactions, tardive dyskinesia, Parkinsonism, akinesia, akathisia, and neuroleptic malignant syndrome.

A patient who is taking a monoamine oxidase inhibitor (MAOI) presents with muscle rigidity, myoclonus, hyperthermia, and cardiovascular collapse. Two days earlier she had been started on another drug. What drug was this patient most likely prescribed? A. Fluoxetine B. Lithium C. Chlorpromazine D. Phenytoin

Fluoxetine This is a typical presentation of serotonin syndrome. The combination of a monoamine oxidase inhibitor and an SSRI can lead to death from excessive serotonin release. Initial treatment requires discontinuation of the medications associated with the syndrome and close monitoring. Hydration and benzodiazepines may be used. Cyproheptadine may be considered in cases refractory to supportive care. Many medications and illicit drugs can cause serotonin syndrome either when taken alone at high doses or in combination with other serotonergic medications.

A 7-week-old boy presents with progressive cough and tachypnea for three days. On examination, the infant is afebrile with rales on pulmonary auscultation. The chest x-ray demonstrates interstitial pulmonary infiltrates. What is the most common adverse reaction of the first-line drug that treats this condition? A. Cholestatic jaundice B. Gastrointestinal discomfort C. Hearing loss D. Maculopapular rash

Gastrointestinal discomfort A patient with interstitial pulmonary infiltrates most likely has atypical pneumonia. Causes of atypical pneumonia include legionella, chlamydia, and mycoplasma. Due to the patient's age, this atypical pneumonia is most likely caused by Chlamydia trachomatis. Chlamydia trachomatis pneumonia is treated with a macrolide, such as erythromycin. The most common adverse effect of erythromycin is gastrointestinal discomfort and nausea. The combination of oral erythromycin and infantile hypertrophic pyloric stenosis has been reported in infants younger than six weeks of age. However, this patient is over six weeks of age, so erythromycin is an acceptable choice.

A pharmaceutical researcher is investigating the role of vitamin D in modulating disease progression by interfering with various intracellular functions. He injects vitamin D as a preparation to different kinds of diseased cells. Which of the following processes is the most likely target the researcher is trying to modulate? A. DNA synthesis B. Protein synthesis C. Gene transcription D. Cholesterol synthesis

Gene transcription Calcitriol is a hydrophobic molecule requiring a carrier protein such as vitamin D-binding protein to be transported in the bloodstream. Calcitriol binds intracellular vitamin D receptors (VDR) found in various types of cells. The activated VDR, in turn, induces transcription activation of specific genes and suppression of others. Intracellular vitamin D receptors (VDR) were discovered in the small intestines, renal tubules, bones, and various types of cell types such as lymphocytes, pancreatic cells, and keratinocytes, parathyroid, and pituitary gland cells. The other processes mentioned are not direct targets for vitamin D.

A 28-year-old male patient with an unremarkable past medical history presents to the outpatient clinic complaining of increased urinary frequency, increased thirst, increased hunger, weight loss, and dizziness. His medications include a daily multivitamin. He denies allergies to food and medications. Physical exam BP 137/89 mmHg, pulse 85 per minute, respiratory rate 18 per minute, rest of the physical exam is unremarkable. Labs are reported as shown: fasting plasma glucose: 180 mg/dL, HbA1C: 7.5%, oral glucose tolerance test: 220 mg/dL, creatinine: 0.9 mg/dL , BUN: 12 mg/dL, GAD-65 antibodies: positive. Which of the following medications is contraindicated in the management of this patient? A. Glipizide B. NPH insulin + rapid-acting insulin C. NPH insulin + regular insulin D. Long-acting insulin + rapid-acting insulin

Glipizide Sulfonylurea medications such as glipizide exert their antidiabetic effect by stimulating insulin release from pancreatic beta cells. Sulfonylureas are used to treat diabetes mellitus type 2. They have no place in the treatment of type 1 diabetes. In contrast to many other oral diabetic medications, sulfonylureas carry a risk of causing hypoglycemia resulting from excessive insulin production, mainly if patient carbohydrate intake is insufficient for any reason. Other commonly prescribed members of this class are glimepiride, which is classified as a third-generation sulfonylurea and glyburide, which is another second-generation sulfonylurea. This patient symptoms and labs are consistent with the diagnosis for type 1 diabetes mellitus. NPH insulin can be started and combined with rapid-acting insulin (lispro, aspart, glulisine). Glipizide is contraindicated in patients with type 1 diabetes. In type 1 DM, the pancreas is unable to secrete insulin, thus using a sulfonylurea to stimulate it is incorrect. NPH and regular insulin is another regimen used to manage these patients. Patients with type 1 diabetes mellitus pathophysiology are the autoimmune destruction of the pancreatic beta cells. GAD-65 antibodies are associated with type 1 diabetes mellitus. Glipizide is a drug for the management of type 2 diabetes mellitus. Contraindicated in HSS, DKA, and type 1 diabetes mellitus. Patients with type 1 diabetes mellitus, can be treated with long-acting insulin once a day and the combination of rapid-acting insulin, to achieve good metabolic control. In this patient, the symptoms, the age, and labs are suggestive of type 1 diabetes mellitus. Diet and exercise are essential aspects in the treatment of diabetes. Glipizide is a sulfonylurea; it can be used in combination with other hypoglycemic drugs, in the treatment of type 2 diabetes. Glipizide is contraindicated in patients with sulfa allergy.

What is the best medication to improve perfusion of the heart six hours after a myocardial infarction? A. Acetylsalicylic acid B. Clopidogrel C. Warfarin D. Alteplase

The primary application of thrombolytic agents, e.g., alteplase, is in emergency use for an acute myocardial infarction (MI). Under ideal conditions (within 12 hours following an MI), these agents will break down the thrombus in the coronary artery and recanalize the vessel. Alteplase is a synthetic version of a naturally occurring tissue plasminogen activator. Alteplase is also used to treat ischemic stroke.

A 48-year-old female patient with a past medical history of hypertension, hyperlipidemia, and obesity presents to the emergency department complaining of being confused, increased urinary frequency, increased thirst, and increased hunger. She was previously diagnosed a couple of years ago with type 2 diabetes. Her medications include: losartan 100mg, metformin 1000mg, and simvastatin 10mg. She is compliant with her medication apparently, as per her daughter. She follows a strict diet and exercises 3 to 5 days a week. Vital signs show sat02: 94% BP 100/65 mmHg, pulse 93 beats per minute, respiratory rate 18 per minute. Physical exam reveals: mucosas are dry, crackles at the base of the right lung, rest of the physical exam is unremarkable. Labs are reported as shown: hemoglobin A1C: 8.9 %, creatinine: 0.9 mg/dL, BUN: 12 mg/dL, AST: 30 U/L, ALT: 30 U/L, WBC: 14,000/mm3, Beta-hydroxybutyrate: Positive, urinalysis: positive for ketones, arterial blood gases: Bicarbonate: 14 mEq/L. Chest x-ray shows the presence of infiltrates in the lower lobe of the right lung. She is admitted, and started on supplemental oxygen, IV normal saline, IV ceftriaxone, and PO Azythromycin. Given her current condition, which of the following will be contraindicated in the management of this patient at this moment? A. High flow 0.9 % normal saline solution B. Insulin drip C. Ceftriaxone and Azythromycin D. Glipizide

Glipizide This patient is currently in diabetic ketoacidosis, given her positive beta-hydroxybutyrate and her low bicarbonate helps establish the diagnosis. The mainstay for the treatment of DKA is high flow 0.9% Normal Saline to hydrate the patient properly and the insulin infusion and then switch to a mix solution as the condition is improving. Dextrose 5% can be added when the patient blood glucose is < 200 mg/dL. Given the fact the patient is in DKA, the primary treatment that will improve her condition is the normal saline hydration and the insulin drip. Insulin drip will need to be continued until the acidosis improves. Checking the level of bicarbonate will be useful in achieving this. Then the patient can be switched to subcutaneous insulin to continue improving her hyperglycemia. When the glucose level is < 200 mg/dL, the bicarbonate > 15 mEq/L, the anion gap is < 12. The patient DKA is most likely secondary to the Pneumonia she developed. Patients with diabetes mellitus are at risk of developing many infections, and pneumonia is one of the most commons. Continuing antibiotics is extremely needed in this patient to help improve her current condition. Glipizide is a sulfonylurea sold under the brand name Glucotrol. The cell depolarizes resulting in the opening of voltage-gated calcium channels. This causes a calcium influx of effecting insulin release from beta cells. Its mechanism of action is partially blocking potassium channels of the beta cells of the pancreas. Glipizide is contraindicated in diabetic ketoacidosis, HSS. This patient diagnosis is DKA secondary to Pneumonia. The appropriate next step will be admitting the patient, starting IV normal saline, and IV insulin drip, checking the K levels, and supplementing with KCL in the case the patient is not in hyperkalemia. Glipizide can be perfectly added to this patient to help her metabolic control once she recovered from this condition. Her hemoglobin A1c is 8.9 %, that indicates that she will benefit from adding a second agent to treat her type 2 diabetes.

A 55-year-old female with type 2 diabetes mellitus presents for follow-up. Her provider made changes to her diabetes regimen during her previous visit. She was on metformin, liraglutide, and repaglinide, and her provider replaced repaglinide with glipizide. The patient misunderstood the instructions and started taking glipizide along with metformin, liraglutide, and repaglinide. Today, she reports episodes of hypoglycemia. Which of the following combinations is most likely contributing to her hypoglycemia? A. Metformin plus repaglinide B. Metformin plus liraglutide C. Glipizide plus repaglinide D. Glipizide plus liraglutide

Glipizide plus repaglinide Sulfonylureas (i.e., glipizide) may be combined with any other oral antidiabetic medication class except meglitinides (i.e., repaglinide). Sulfonylureas stimulate insulin secretion from the pancreatic beta cells regardless of serum glucose levels. Repaglinide also stimulates insulin secretion from the pancreatic beta cells, but the insulin release is glucose-dependent. The combination of glipizide and repaglinide may have an enhanced hypoglycemic effect leading to hypoglycemic episodes. Metformin and liraglutide are unlikely to cause hypoglycemia. Therefore, the combination of metformin, liraglutide, and glipizide may be used.

A 76-year-old female patient has a history of Type 2 diabetes mellitus and hypertension, and consistently experiences hunger. The only symptom she has is constant sweating. Upon reviewing her chart, her only medications are metoprolol tartrate and a diabetic medication, but she can't remember the name of it. Which of the following medication would most likely cause her symptoms? A. Acarbose B. Glipizide C. Metformin D. Glyburide

Glyburide Glyburide has twice the risk of hypoglycemia compared to glipizide. Metformin and acarbose do not cause hypoglycemia. Diabetes mellitus in the elderly requires special considerations due to impaired renal function and the danger of hypoglycemia. The majority of symptoms of hypoglycemia are masked by beta-blocker medications, such as metoprolol. The only symptom not masked is sweating. Glyburide has the highest risk of causing hypoglycemia, especially in this elderly patient.

A 40-year-old woman, gravida 2 para 2, comes to the office for the complaints of frequent urination, dry mouth, and fatigue for the last 6 months. Her medical history is significant for gestational diabetes in her previous pregnancy 6 years ago, which was managed appropriately with nutritional therapy, exercise, and glyburide. Her menstrual periods are regular, occurring every 28 days with 3-4 days of moderate bleeding. She is sexually active with her husband and desires to get pregnant. Family history is notable for diabetes mellitus type 2 in her mother. Her temperature is 37 C (98.6 F), blood pressure is 137/87 mmHg, the pulse is 77/min, and respirations are 16/min. Her BMI is 32 kg/m2. Further evaluation confirms the diagnosis of type 2 diabetes mellitus, and several oral hypoglycemic agents (OHAs) are discussed. She shares her preference for glyburide due to her past experience with it. Which of the following is the most appropriate advice regarding OHAs, including glyburide, for this patient? A. Patients with pregestational diabetes can continue taking oral hypoglycemic agents throughout pregnancy B. Patients taking oral hypoglycemic agents do not face any risk of hypoglycemia C. Patients with pregestational diabetes should not be taking oral hypoglycemic agents during pregnancy D. Patients taking oral hypoglycemic agents irregularly can become allergic to insulin, so following a regular schedule is critical

Glyburide is a second-generation oral hypoglycemic agent (OHA) from the class of sulfonylureas and is classified as an insulin secretagogue. It, along with other oral hypoglycemic agents, forms the basis of management of type 2 diabetes mellitus. In contrast, some selected OHAs, which include metformin and glyburide, are considered first-line in the management of gestational diabetes. However, it is generally preferred to achieve optimal glycemic control on intensive insulin therapy throughout pregnancy in women with pregestational diabetes, both type 1 and type 2. This is ideally done before conception so that the critical time of organogenesis is covered by appropriate glycemic control. Therefore, these drugs are discontinued, and insulin therapy is initiated. However, if a patient has good glycemic control on an OHA (e.g., metformin), it is generally advised to continue taking that drug while being shifted to insulin, and its levels have become sufficient for normal glycemic control. Regular meals, activity, and rest are crucial while taking oral hypoglycemic agents to have normal glycemic control. Blood glucose levels may fluctuate while on OHAs, with hypoglycemia being a very troublesome adverse-effect of sulfonylureas. Patients need to know the signs of both hypoglycemia and hyperglycemia for earlier identification and management of such side-effects. Oral hypoglycemics are not an insulin substitute and are not associated with the development of possible allergy to insulin.

Aminoglycosides are indicated for the empirical treatment of any diseases such as infectious endocarditis and sepsis. They are often utilized in combination therapy as well. However, when used alone they are effective in treating infections caused by Yersinia pestis, Francisella tularensis, and Campylobacter. Based on infections aminoglycosides can be used as monotherapy. What is the major therapeutic target of aminoglycoside antibiotics? A. Gram-positive organisms B. Gram-negative organisms C. Anaerobes D. Fungal pathogens

Gram-negative organisms Aminoglycosides have excellent activity against gram-negative bacteria. Yersinia, Campylobacter, and Francisella are all gram-negative pathogens. These antibiotics have been used less in the past decade because of their side effects of ototoxicity and nephrotoxicity. Aminoglycosides are often used in combination with antibiotics with activity against bacterial cell walls, such as beta-lactams and vancomycin, for broader coverage. Aminoglycosides are effective in the empirical treatment of systemic gram-negative infections but levels must be monitored to prevent toxicity.

An adult male is brought to a clinic with bizarre thoughts and actions. The client has hallucinations and delusion affecting his quality of life for more than 6 months now. After an evaluation, the provider diagnoses schizophrenia and plans to start the client on a psychotropic agent. What psychotropic agent is associated with the highest incidence of extrapyramidal side effects? A. Thioridazine B. Olanzapine C. Risperidone D. Haloperidol

Haloperidol Extrapyramidal side effects are caused by antipsychotic medications. While side effects are more common with the more potent traditional antipsychotics like haloperidol or fluphenazine, they can and do occur with the atypical antipsychotics and with low potency traditional antipsychotics (e.g., Chlorpromazine, loxapine, and thiothixene). The side effects can include Parkinsonian tremors, muscular rigidity, akathisia, and akinesia or a lack of movement. The patient's facial movements may be so restricted as to appear as if the patient was wearing a rigid mask. There might be sudden muscle spasms of the head, neck, limbs, or trunk.

An 8-year-old girl comes in with her mother. She has always done well in school, but recently her teachers have been complaining that she has been inattentive and performing poorly. She still performs well in written tests. Her medical history is significant for a urinary tract infection seven months ago, which was complicated by hydronephrosis. She was admitted and given an IV medication that successfully treated the infection. Family history is insignificant. What is the most likely cause of her declining progress at school? A. Persistent urinary tract infection B. Renal failure C. Hearing loss D. Attention deficit hyperactivity disorder

Hearing loss This child was performing well and is still doing well in written tests. This means her attentiveness is related to her not hearing her teacher well. There has been a recent UTI treated with an antibiotic. Aminoglycosides are used to treat UTIs and can lead to hearing loss. Aminoglycoside-induced ototoxicity has been reported to occur in 2 to 45% of adults. The ototoxicity can be vestibular and/or cochlear and is typically dose-dependent. Gentamicin, streptomycin, and tobramycin more commonly cause vestibular damage, while amikacin and kanamycin result in more cochlear damage. Studies have found that aminoglycosides seem to create reactive oxygen species within the inner ear; this, in turn, causes damage to the vestibular and cochlear sensory cells along with cochlear neurons. Often the vestibular loss is salvageable, while hearing loss is irreversible. For toxicity purposes, renal function and cochlear function require monitoring. Serial audiometry may be considered to prevent irreversible hearing loss. There is no antidote for the toxicities of aminoglycosides. However, agents with protective effects on the ear and kidney may help prevent aminoglycoside-induced toxicity. In particular, N-acetylcysteine demonstrates promising protective effects on patients using aminoglycoside. ADHD will lead to other symptoms, along with poor performance in school. Renal failure and persistent UTIs would not cause a decline in academic performance in this patient.

A 16-year-old girl is brought to the clinic for follow-up. She has a history of partial epilepsy and has been poorly controlled with first-line agents. The clinician decides to add a novel antiepileptic agent that functions by blocking the T-type calcium channels with some action on sodium channels. This drug has a weak carbonic anhydrase inhibiting action as well. Which of the following is the strongest contraindication to treatment with this medication? A. Current pregnancy B. Hypersensitivity to sulfa drugs C. Bone marrow suppression D. Visual disturbances

Hypersensitivity to sulfa drugs This patient has refractory partial epilepsy. The drug to be added is zonisamide. It is an adjunct to first-line agents for the treatment of partial seizures. It belongs to the sulfonamide class of drugs. Zonisamide is contraindicated in individuals with hypersensitivity to sulfonamides. Severe reactions such as Stevens-Johnson syndrome (SJS), toxic epidermal necrolysis (TEN), and fulminant hepatic necrosis have been reported with zonisamide use. Considerations should be given to stopping the drug in individuals who develop a skin rash. Close monitoring of the patient's condition is required if the drug is not discontinued. Zonisamide is a pregnancy category C drug and is not absolutely contraindicated in pregnancy. Carbamazepine is contraindicated in patients with marrow suppression, and vigabatrin should be used with caution in a patient with preexisting visual disturbances.

A 72-year-old patient with a history of hypertension and hypothyroidism is brought to the emergency department by the EMS. The client had been experiencing headaches for the last few days, but it became worse today. She is afebrile, has a heart rate of 90 beats/min, bood pressure of 208/100mm, and a respiratory rate of 18/min. She describes her vision as "hazy." The healthcare provider prescribes 40 mg of intravenous push nitroprusside. If the nitroprusside comes in 20mg/mL vials, then how much of the drug show the nurse administer? A. 1mL B. 2mL C. 4mL D. 6mL

Hypertensive emergency is defined as the presence of high blood pressure with resulting end-organ dysfunction, e.g. headache, blurry vision, or dyspnea. Sodium nitroprusside is the first-line treatment for hypertensive emergency. Sodium nitroprusside decreases the blood pressure by decreasing the afterload via arteriolar vasodation. It also decreases the preload by large vein smooth muscle relaxation.Medication dosages can be calculated by the desired over have method, which allows for the calculation of an unknown quantity of the drug to be administered: (Ordered dose amount divided by the amount avalable) multiplied by (quantity) = Desired amount to administer (x).The correct answer is 2mL, i.e. (40 mg of ordered dose/20mg available) x (1mL quantity) = 2 ml

A 48-year-old male presents to the emergency department due to a high-grade fever and chills. He also reports shortness of breath and cough, starting two days before the onset of the fever and chills. He denies any illicit drug or significant alcohol use. The patient has smoked one pack of cigarettes daily for 20 years. On examination, his temperature is 39.5 C (103.1 F), pulse 102/min, respirations 27/min, and blood pressure 100/65 mmHg. Lung examination reveals dullness to percussion on his right side. Chest x-ray shows pulmonary infiltrates, and Gram stain of the sputum shows gram-positive diplococci. A slow intravenous infusion of levofloxacin 750 mg is ordered over 90 minutes. Which of the following is likely to be prevented by using slow infusion of the antibiotic rather than rapid infusion? A. Hypotension B. QT interval prolongation C. Crystalluria D. Tendon rupture

Hypotension Levofloxacin, a broad-spectrum antibiotic, can be used against both penicillin-sensitive and resistant, gram-positive organisms. It is also called a respiratory fluoroquinolone because of its excellent lung tissue penetration and higher activity against most common respiratory pathogens, including Streptococcus pneumoniae. Levofloxacin produces bactericidal effects by interfering with bacterial DNA synthesis. It does so through inhibition of DNA-gyrase in susceptible organisms and thereby promoting the breakage of DNA strands leading to the relaxation of supercoiled DNA. Due to the growing concern of drug resistance to fluoroquinolones, it should be used only for strongly suspected bacterial infections. Due to its high oral bioavailability (99%), levofloxacin can be administered interchangeably through oral and intravenous routes. However, intravenous administration should be given by slow infusion over 60 minutes (250-500 mg) or 90 minutes (750 mg) due to the increased risk of hypotension associated with rapid infusion. Besides, levofloxacin should not be infused in a solution containing multivalent cations. Providers should instruct patients to drink adequate amounts of water during the intake of oral levofloxacin to prevent crystalluria. Before administering this antibiotic, the provider should screen patients for a history of QT interval prolongation and myasthenia gravis. Due to the risk of tendon rupture and tendinitis, levofloxacin should be limitedly used in uncomplicated urinary tract infection, acute bacterial sinusitis, and acute bacterial exacerbation of chronic bronchitis.

A 78-year-old woman is brought to the hospital with severe left-sided low backache. She was recently diagnosed with stage 4 breast cancer, including metastasis to the L4 vertebra. She has chosen palliative care and has been using hydromorphone and acetaminophen for pain; however, it has not been helpful for this episode. She is holding her back in pain. Vital signs are significant for a heart rate of 100/min. Which of the following is the next best step in the management of this patient? A. Therapeutic chemotherapy B. Epidural analgesia C. IV morphine D. Palliative radiotherapy

IV morphine The patient has presented with a painful spinal metastasis. Palliative radiotherapy is a suitable treatment for this patient but will take time to have an effect. Morphine should be started to relieve her pain.The precise mechanism of action of radiotherapy in the treatment of metastasis is not known. However, it is believed that it causes tumor shrinkage and decreased inflammation.Another appropriate treatment might be IV corticosteroids.Since the patient has chosen palliative care, therapeutic chemotherapy is an inappropriate choice. Epidural analgesia may be difficult to perform for this patient and associated with more complications. Since the patient has already received two doses of a strong opioid, adding morphine would not be appropriate or helpful.

A 22-year-old woman is admitted to the burn floor to manage 3rd-degree burns covering 18% of her body. She was the victim of an attack where the assailant threw boiling water on her. The patient presented to the emergency department for stabilization. After the irrigation of her wounds, silver sulfadiazine is chosen as her prophylactic topical antibiotic. It is decided that the patient will require a skin graft. What is the most appropriate time to stop the application of the topical agent before skin graft placement? A. 48 hours in advance B. 24 hours in advance C. 12 hours in advance D. Immediately before grafting

Immediately before grafting Silver sulfadiazine should cover the wound completely at all times. Silver sulfadiazine should be applied one or two times daily with or without a dressing covering the ointment. Using the sterile technique, a layer 1/16" should be applied to the entire burn. Topical silver sulfadiazine should be used until the wound is completely healed or the skin is ready to be grafted.

A 67-year-old non-smoker, non-alcoholic male patient visits the office with his wife complaining of dark urine and abdominal pain since a week. His wife has noticed him being more tired than usual for a month. His medical history includes ddiabetes mellitus type 2 and hypothyroidism. His current medications include thyroxine, pioglitazone, and metformin. On examination, his vitals and system review are within normal limits, except for a yellow sclera. Lab reports demonstrate deranged liver function tests. If the cause of the patient's symptoms is drug-induced, what is the mechanism of action of the potentially responsible drug being used by the patient? A. Increase insulin sensitivity B. Increase insulin secretion C. Decrease insulin secretion D. Block insulin receptors

Increase insulin sensitivity The thiazolidinediones e.g. pioglitazone, a class of anti-diabetic medications, used for diabetes mellitus type 2, acts on PPAR-gamma receptors to increase insulin sensitivity. A known side effect of Pioglitazone is to cause deranged liver function tests(LFT). LFTs are performed before, and at follow up every 2 months for the first year and then periodically, after starting therapy. Patients should be advised to look for signs like unexplained fatigue, abdominal pain, dark urine, etc and report to their doctor when on this drug. Pioglitazone does not promote the release of insulin from the pancreas. Use of pioglitazone results in a lowering of hyperglycemia and hypertriglyceridemia, as well as improvements in HbA1c.

A 75-year-old female is startled by a cat running across her path at night while taking a walk. In response to this situation, the chromaffin cells secrete a hormone into the bloodstream. Although this chemical targets multiple systems and has various effects, what is its effect intracellularly on the heart? A. Increased cyclic adenosine monophosphate (cAMP) concentrations B. Increased cyclic guanosine monophosphate (cGMP) concentrations C. Increased inositol triphosphate (IP3) concentrations D. Increased diacylglycerol (DAG) concentrations

Increased cyclic adenosine monophosphate (cAMP) concentrations Targeted activation of the beta 1 receptor in the heart by epinephrine increases sinoatrial nodal, atrioventricular nodal, and ventricular muscular firing, thus increasing heart rate and contractility by increasing intracellular concentrations of cAMP. As either stroke volume or heart rate increases, both of which will increase with targeted activation of the beta-1 receptor, cardiac output will increase, thus increasing perfusion to tissues throughout the body. Various hormones may target adrenoreceptors with different affinities. Epinephrine, dopamine, and isoproterenol target beta-1 and beta-2 receptors. Beta-1 receptors, along with beta-2, alpha-1, and alpha-2, are adrenergic receptors primarily responsible for signaling in the sympathetic nervous system.

A 17-year-old male patient presents to the emergency department after his friends found him unconscious lying face down at a party. He has a past medical history of panic disorder. His respiratory rate is 7/min, blood pressure 92/70 mmHg, the temperature is 98.0 F (36.7 C), and heart rate 64 beats/min. The patient is unable to cooperate with a physical examination. The physician administers flumazenil, which improves his condition within minutes. His respirations are now 13/min, blood pressure is 110/82 mmHg, pulse 78/min, and temperature 98.0 F (36.7 C). What is the mechanism of action of the drug this patient has most likely ingested? A. Activation of mu-opioid receptors B. Increased duration of chloride channel opening C. Increased frequency of chloride channel opening D. Inhibition of serotonin reuptake

Increased frequency of chloride channel opening The patient likely ingested an overdose of a benzodiazepine, which caused severe respiratory depression, as well as a loss of consciousness. Flumazenil is effective for diagnostic and therapeutic reversal of benzodiazepine receptor agonists. Flumazenil has a rapid onset of action in 1 to 2 minutes. Flumazenil injection is indicated for a complete or partial reversal of the sedative effects of benzodiazepines in conscious sedation and general anesthesia in the adult and pediatric populations. Flumazenil speeds the recovery from sedation following minor surgical procedures and shortens the post-operation monitoring period for minor surgery, resulting in earlier patient discharge. Flumazenil can precipitate seizures, therefore it should be used judiciously. Consider observation alone in a patient who has good respiratory exchange.

A 61-year-old woman presents to the emergency department in a confused state. The patient's daughter called 911 when her mother became unresponsive to conversations with her. The daughter also notes that the patient was recently diagnosed with community-acquired pneumonia. Lab values are the following: plasma glucose = 625 mg/dL, arterial pH = 7.35, sodium bicarbonate = 19 mEq/L, serum ketone = small, and serum osmolality = 325 mOsm/kg. Which of the following would provide the best management for this patient? A. Intravenous insulin B. Subcutaneous insulin C. Intravenous glucagon D. Subcutaneous glucagon

Intravenous insulin Insulin is used to treat hyperglycemia. Insulin can cause hypoglycemia. Therefore, patients with diabetes mellitus need to carefully monitor their plasma glucose concentrations. Insulin is commonly administered subcutaneously and it can also be administered intravenously for the treatment of severe hyperglycemia because intravenously administered insulin has a faster onset of action as well as ease of titration. Glucagon will not treat the patient's hyperglycemia. Subcutaneously administered insulin has a slower onset of action.

A 16-year-old female patient with a long history of irritable bowel syndrome continues to have symptoms of pain, bloating, and diarrhea despite extensive evaluation and trials of fiber supplements, loperamide, and imipramine. The patient is started on alosetron, 0.5 mg twice a day. A week later, she presents to the emergency department with mild to moderate periumbilical abdominal pain and hematochezia. Vital signs are blood pressure 90/50 mmHg, heart rate 120 beats/min, and respiratory rate 19 breaths/min. Her abdomen is tender in the periumbilical region. A rectal exam shows no masses or tenderness but is heme positive. Labs show white blood cell count of 15,000 cells/microliter with 75 percent polymorphonuclear neutrophils (PMNs) and slight anion gap metabolic acidosis. What is the most likely diagnosis? A. Gastric ulcer with perforation B. Appendicitis C. Ectopic pregnancy D. Ischemic colitis

Ischemic colitis Alosetron is a serotonin receptor antagonist that reduces the perception of painful visceral stimulation and induces rectal relaxation. It was withdrawn from the market in 2000 due to episodes of ischemic colitis, which were rarely fatal. It was re-released with a black box warning and strict monitoring program. The patient should have a CT of the abdomen and pelvis. Management is supportive, but surgery may be needed.

A 25-year-old male is having a blood transfusion due to massive blood loss secondary to a road traffic accident. His vitals before starting the transfusion are pulse rate 72/min, blood pressure 122/75 mmHg, respiratory rate 16/min, and temperature 98.6F (37 °C). During the transfusion, he develops fever and flank pain. The transfusion is immediately stopped. What is the next step in the management of this patient? A. Insert a Foley catheter B. Fluid restriction C. O.1N HCL solution D. Isotonic fluids and furosemide

Isotonic fluids and furosemide Whenever a blood transfusion reaction is suspected, the blood transfusion must be stopped immediately. The next step is to start isotonic fluids. The flank pain in this patient points to renal involvement. This may indicate an acute hemolytic transfusion reaction. It involves hemolysis of transfused red cells due to the presence of preformed antibodies. The patient should be hydrated, and furosemide should be administered. Diuretics can help maintain renal blood flow and preserve urine output. Sometimes low dose dopamine may be required to maintain renal blood flow and blood pressure. Close monitoring is required in this case, and evaluation of renal functioning should be done to avoid acute kidney injury.

A 65-year-old patient presents to the healthcare provider for a routine check-up. He has a past medical history of diabetes mellitus type 2 and hypertension. His father had died after a stroke at the age of 70, and his mother had a myocardial infarction at 66. Labs reveal cholesterol of 245 mg/dl, an LDL of 150 mg/dl, and triglycerides of 170 mg/dl. The provider decides to begin prescribing medication to address the abnormalities and lower his risk for adverse cardiovascular events. Which of the following should be monitored in this patient? A. Renal function B. Pulmonary function C. Liver enzymes D. Cardiac enzymes

Liver enzymes Baseline levels of lipids, creatine kinase, and liver functions should be obtained in all individuals before starting statins. Liver function tests should be assessed within 4-6 weeks after starting therapy. Treatment should be discontinued if serum transaminase levels rise. All patients on statin therapy should be advised to report any muscle pain. The development of myositis, myalgia, and myopathy is rare but does occur. Myopathy presents with marked elevations of creatine kinase and muscle pain.

A 71-year-old with an acute myocardial infarction needs urgent surgery to repair his mitral valve. His blood pressure is low. Which of the following anesthetics would be very useful in this patient? A. Ketamine B. Succinylcholine C. Halothane D. Propofol

Ketamine causes dissociative anesthesia and is a very useful analgesic agent. Unlike most anesthetics, it does not cause depression of blood pressure and is useful in patients with a weak heart. Ketamine can increase heart rate and systolic blood pressure during induction. Ketamine does not depress respiration.

In cases of type 2 diabetes, when diet and exercise have failed to bring about adequate glycemic control, metformin is commonly initiated as a first-line oral therapy. Which of the following is true about metformin? A. Hypoglycemia is a common side-effect B. Lactic acidosis is the most concerning side-effect C. Increases pancreatic beta-cell insulin production D. Meglitinide class of drugs

Lactic acidosis is the most concerning side-effect Metformin is a medication in the biguanide class and is the most common first-line agent used to treat and manage type 2 diabetes mellitus. While it is generally well-tolerated, in cases of liver dysfunction or large doses, lactic acidosis is a relatively uncommon but potentially serious side effect. It has a very low incidence of causing hypoglycemia, which is a side effect more commonly seen in the sulfonylurea class of diabetes medications. Metformin works by decreasing liver glucose production, as well as increasing somatic tissue sensitivity to insulin.

A 24-year-old construction worker presents to the emergency department after a fall 30 minutes before arrival. He fell on some broken glass and sustained a laceration to his left forearm. 1% lidocaine is combined with an adjunctive medication to anesthetize the wound and reduce bleeding before exploring the injury. What is the mechanism of action of this adjunctive medication? A. Local vasodilation B. Local vasoconstriction C. Blocks sodium channels D. Blocks potassium channels

Local vasoconstriction Lidocaine has intrinsic vasodilatory effects, increasing local blood flow when injected into a wound. Adding low-dose epinephrine counteracts this by causing mild vasoconstriction, allowing anesthesia to be achieved while minimizing blood loss. The vasoconstrictor effects of epinephrine also reduce the rate at which lidocaine is removed from the site of the procedure through the vasculature, meaning that anesthesia should be effective for longer. When given with epinephrine, the maximum safe dose for lidocaine increases from 3 mg/kg to 7 mg/kg. Systemic exposure to excessive lidocaine results in the central nervous system (CNS) and cardiovascular effects. CNS effects occur at lower blood plasma concentrations, and additional cardiovascular effects present at higher concentrations, though cardiovascular collapse may still occur with low concentrations.

A 44-year-old man complains of excessive shivering after having a partial gastrectomy procedure. Vital signs show a blood pressure of 110/82 mmHg, a pulse of 84/min, a respiration rate of 17/min, and an oxygen saturation of 97%. On physical examination, he is shivering uncontrollably. The healthcare provider then administers a synthetic opioid that alleviates the patient's symptoms. The administered medication acts on which of the following receptors to alleviate this patient's symptoms? A. K-opioid receptors B. Mu-opioid receptors C. Delta-opioid receptors D. Mu1-opioid receptors

Meperidine acts on the k-opioid receptors to decrease shivering in patients through a poorly understood mechanism. This patient was most likely administered meperidine to treat his postanesthetic shivering after his partial gastrectomy surgery. Meperidine also stimulates mu-opioid receptors to produce analgesia, but this is not the receptor that controls shivering, as seen in this patient.Meperidine does not act in the delta-opioid receptors.

A 65-year-old woman presents to the emergency department due to a recent fall in the street. She was jogging in a neighborhood park when she tripped and fell on her outstretched hands. The patient has a history of hypertension and is currently taking lisinopril. Vital signs are a blood pressure of 124/82 mmHg, a pulse of 81/min, and a respiratory rate of 17/min. On physical examination, she has tenderness on her right wrist with visible deformity. An x-ray shows a Colles fracture. She is administered meperidine to treat her pain. This medication belongs to which of the following classes of drugs? A. Anxiolytic B. NSAID C. Corticosteroid D. Opioid analgesic

Meperidine is a synthetic opioid pain reliever.It is indicated for the treatment of moderate to severe pain.It exerts its analgesic effect via several mechanisms in the same manner as morphine, principally affecting the central nervous system and smooth muscle tissue.There is evidence that meperidine may affect smooth muscles less than other members of the opioid analgesic class, resulting in a lower incidence of opioid-induced constipation.

A 55-year-old man with a past medical history of diabetes mellitus complicated with gastroparesis is scheduled for hiatal hernia surgery. The healthcare provider advised metoclopramide and famotidine in preoperative prophylaxis for nausea and vomiting. Which of the following is the effect of the antiemetic on gastric volume and pH? A. Metoclopramide decreases gastric volume but increases gastric pH B. Metoclopramide decreases gastric volume but has no effect on gastric pH C. Metoclopramide increases gastric volume but has no effect on gastric pH D. Metoclopramide has no effect on gastric volume but it decreases gastric pH

Metoclopramide decreases gastric volume but has no effect on gastric pH Metoclopramide is a prokinetic drug. It increases gastric emptying and thus decreases gastric volume. Metoclopramide is a dopamine receptor agonist, and it can cause extrapyramidal side effects. Metoclopramide also increases lower esophageal sphincter tone but has no effect on gastric pH. Ten mg metoclopramide has not been found effective for nausea and vomiting. Higher doses are needed for decreasing postoperative nausea and vomiting. Famotidine decreases gastric pH. It is histamine H2-receptor antagonists (H2 blocker).

A medical student is involved in a trial involving the interaction of the drug codeine with opioid receptors in the body. The drug has an agonistic effect on the opioid receptor. Pain relief is primarily achieved by the action of this drug on opioid receptors in which organ? A. Cerebral cortex B. Midbrain C. Medulla D. Spinal cord

Midbrain Within the nervous system, activation of mu receptors in the midbrain is considered as the primary mechanism of opioid-induced analgesia. Codeine is an effective cough suppressant and is also addictive. The cough reflex is primarily mediated through the opioid receptors present in the medulla. It is a pure agonist and does not possess any antagonistic properties. Respiratory depression can occur with large doses of this opioid antitussive.

A 65-year-old female is discharged from the hospital after knee replacement surgery. The clinician prescribed her medicine for pain, which acts as a partial agonist on opioid receptors. Which of the following medications should be avoided while taking this pain medicine? A. NSAIDs B. Cephalosporins C. Monoamine oxidase inhibitor D. Lithium

Monoamine oxidase inhibitor Tramadol is an opioid, and like other opioids, selectively binds to different opiate receptors in the central nervous system. It has a partial agonist activity on the opioid receptors; it also inhibits the reuptake of serotonin and norepinephrine. Concomitant use of monoamine oxidase inhibitor (MAOI) with the tramadol results in increasing serotonin levels and can result in serotonin syndrome. Serotonin syndrome is a potentially life-threatening condition precipitated by the use of serotonergic drugs. MAOIs inhibit the breakdown of the neurotransmitters, including serotonin and, thus, increasing their levels. Patients currently on MAOIs or people who have been on MAOIs in the past fourteen days should not receive tramadol. Most cases of serotonin syndrome are not severe and will resolve with removal of the offending drug alone. After stopping all serotonergic drugs, management is largely supportive and aimed at preventing complications.

A 16-year-old male with a history of asthma presents to the urgent care with an asthma exacerbation secondary to an upper respiratory infection. His daily medications are a short-acting beta-agonist to control his daily allergies. He reports that he is using his albuterol (short-acting beta agonist) daily. On physical examination, he has diffuse bilateral wheezing bilaterally, and he is not using accessory muscle. The provider adds a montelukast to control his asthma symptoms and allergies. What is the mechanism of action for this drug? A. Beta-adrenergic agonist B. Leukotriene antagonist C. Reduces mast cell degranulation D. Phosphodiesterase inhibition, leading to reduced cAMP metabolism

Montelukast is a leukotriene antagonist. It blocks the action of leukotriene on cysteinyl leukotriene receptors. Leukotrienes cause bronchoconstriction and are pro-inflammatory mediators. Montelukast has very few drug-drug interactions, even compared to zafirlukast, another leukotriene inhibitor given for asthma treatment.

A 52-year-old woman presents for a regular check-up. She has a past medical history of type 2 diabetes and hypertension. She reports that over the past month, she has experienced bloating, early satiety, and loss of appetite. Over the past week, she has felt nauseous and had episodes of vomiting undigested food. She denies alcohol, tobacco, or recreational drug use. She is currently taking metformin and lisinopril, but her blood sugars have been labile. Her current hemoglobin A1C is 10.0% (4-6). What is the mechanism of action of the antibiotic that can treat this condition? A. Inhibition of DNA synthesis B. Inhibition of cell wall synthesis C. Motilin receptors binding D. Inhibition of 30S ribosome

Motilin receptors binding This patient with bloating, early satiety, loss of appetite, nausea, and vomiting, along with uncontrolled diabetes, likely has diabetic gastroparesis. Erythromycin is an antibiotic used to treat diabetic gastroparesis. Erythromycin binds to motilin receptors. These are members of the G protein-coupled family. Motilin is an intestinal peptide that stimulates smooth muscle contraction in the gut and increases gut motility. While erythromycin has the ability to inhibit the 30S ribosome, the specific mechanism of action related to the treatment of diabetic gastroparesis is through the binding of motilin receptors.

A 21-year old man is brought to the emergency department in an unconscious state. The patient is unresponsive with a pulse rate of 90/min, blood pressure of 100/60 mm Hg, and a respiratory rate of 6/min. His pupils are constricted and nonreactive to the light stimulus. His friends report that he has been into recreational drug abuse. The provider decides to administer a drug to help him. Where would this drug act? A. Mu-opioid receptors B. Cholinergic receptors C. Histamine receptors D. GABA receptors

Mu-opioid receptors Naloxone is an opioid antagonist used to treat an overdose from opioids such as morphine or heroin. Naloxone works very rapidly and blocks the opioid receptors competitively. Specifically, it has the greatest activity at the mu-opioid receptor. Within 1 to 2 minutes of administration intravenously or intraosseously, most patients with a morphine overdose can be reversed. Besides intravenous or intraosseous routes, naloxone can be administered intranasally, intramuscularly, subcutaneous, or inhaled via nebulizer.

A 65-year-old male presented to the healthcare provider two weeks after experiencing an anterior wall myocardial infarction. He has no past medical history of coronary artery disease or smoking and only has elevated blood pressure that is well-controlled with lisinopril. The healthcare provider counsels the patient on dietary modifications and exercise. He has also prescribed a drug to manage his hypercholesterolemia. What is the side effect of the most significant concern with the administration of this drug? A. Constipation B. Impaired hearing C. Impotence D. Myopathy

Myopathy Hydroxymethylglutaryl-CoA (HMG-CoA) reductase inhibitors, or statins, are a class of drugs that lower total cholesterol, low-density lipoprotein (LDL), and triglyceride concentrations while increasing high-density lipoprotein (HDL) concentrations. Lovastatin is naturally occurring in oyster mushrooms and red yeast rice. As with many statins, drinking grapefruit juice during therapy increases side effects, as grapefruit juice inhibits the metabolism of statins. Lovastatin can cause rhabdomyolysis with acute renal failure. Thus, all patients should be closely monitored for muscle pain. The drug should be discontinued before any elective surgery.

A 16-year-old female presents to the emergency department with complaints of nausea, vomiting, and abdominal pain after she ingested 30 pills of an unknown drug 6 hours ago. Her family does not know what kind of pills she took. She was given intravenous sodium bicarbonate, but her symptoms did not resolve. Which of the following is responsible for her condition? A. N-acetyl benzoquinone imine (NAPQI) B. Aspirin (acetylsalicylic acid) C. Cocaine D. Morphine

N-acetyl benzoquinone imine (NAPQI) Acetaminophen is subsequently metabolized to NAPQI by cytochrome P450. NAPQI is a toxic substance that is safely reduced by glutathione to nontoxic mercaptate and cysteine compounds, which are then renally excreted. Administration of sodium bicarbonate increases the urinary excretion of organic acids. Acetaminophen is neither a base nor an acid; hence its excretion is unaffected by urine pH. Metabolism of acetaminophen primarily occurs through glucuronidation and sulfuration, both of which occur in the liver. In an overdose, these pathways are saturated, and more acetaminophen is subsequently metabolized to NAPQI by cytochrome P450. Morphine, aspirin, and cocaine are all organic alkali compounds; therefore, alkalization of the urine will increase their urinary excretion.

A 34-year-old male comes to the emergency department with altered mental status. He is accompanied by his friend, who said he consumed something to relax. His blood pressure is 120/90 mmHg, the pulse is 65/min, respirations are 8/min, and the temperature is 99 F. Physical examination reveals altered sensorium with decreased pupil size. The patient's history is insignificant except for occasional cannabis use. Family history is noncontributory. Which of the following is the most likely treatment option for this patient? 1. Flumazenil 2. Naloxone 3. Methadone 4. Fomepizole

Naloxone Opioid use disorder is the chronic use of opioids that causes clinically significant distress or impairment. The etiology of opiate use disorder is multifaceted. Dependence and substance abuse is a product of biological, environmental, genetic, and psychosocial factors.Opioid intoxication symptoms include confusion, miosis, hypersomnia, nausea, euphoria, constipation, and decreased pain perception. Naloxone is indicated for the treatment of opioid toxicity, specifically to reverse respiratory depression from opioid use. It is useful in accidental or intentional overdose and acute or chronic toxicity. Common opioid overdoses treated with naloxone include heroin, fentanyl, carfentanil, hydrocodone, oxycodone, methadone, etc.Naloxone is a pure, competitive opioid antagonist with a high affinity for the mu-opioid receptor, allowing for reversal of the effects of opioids. The onset of action varies depending on the administration route but can be as fast as one minute when delivered intravenous (IV) or intraosseous (IO). It has a half-life of 30 to 120 minutes, depending on the administration route, with IV being the fastest. In most cases of overdose, the reversal is observable in a matter of minutes. If no signs of reversal are apparent after administration of 10 mg of naloxone, one must consider co-ingestants like tricyclic antidepressants, alcohol, benzodiazepines, cocaine, methamphetamine, and club drugsDiazepam is a benzodiazepine that will aggravate respiratory depression. Methadone, an oral mu agonist, is commonly used in opioid replacement. It has been widely used and studied worldwide, and methadone maintenance is a well-established approach. Fomepizole is used in ethylene glycol poisoning.

A 16-year-old trauma patient has an epidural infusion of morphine before his knee surgery. Three hours later, he reports itching, and his respiratory rate is four breaths per minute upon assessment. He opens his eyes when you speak to him for a couple of seconds and then immediately falls back asleep during your interview. Which of the following medications is the most appropriate treatment for this patient? A. Naloxone B. Oxygen C. Diphenhydramine D. Intravenous fluids

Naloxone is an opioid antagonist often used to counter the respiratory depression following morphine overdose. Naloxone has an affinity for the mu-receptors and rapidly reverses the effects of morphine. Naloxone is commonly administered intravenously but can also be given via intramuscular or intranasal routes. A small initial dose, usually 0.04 to 0.1mg IV is recommended in opioid-dependent patients with symptoms of opioid overdose to avoid opioid withdrawal symptoms. Larger initial dosing, such as 0.4mg IV, is recommended in patients without a known history of opioid dependency.

A 43-year-old male is brought to the office by his wife regarding his alcohol use. The patient states that he starts with one shot but then loses control and ends up drinking 7 or 8 shots daily. The wife says that this has been the cycle for the past three years. Which of the following drugs is FDA approved for long-term treatment of a patient suffering from alcohol use disorder? A. Lithium B. Diazepam C. Naltrexone D. Chlorpromazine

Naltrexone Naltrexone, acamprosate, and disulfiram are FDA approved for the long-term treatment of a patient suffering from alcohol use disorder. Naltrexone can reduce cravings for alcohol. It is given to individuals only after they have abstained from alcohol for 1 to 2 weeks. It is not used during detoxification from alcohol dependence as it is not as effective. It can help some people suffering from alcohol use disorder as a part of their long-term treatment plan.

A 35-year-old male with a history of herpes simplex virus 1 and herpes simplex virus 2 infection presents to the clinic with targetoid lesions as a result of central epidermal necrosis with surrounding erythema on his hands. The physical examination shows no other lesions on the rest of his body. Histological examination shows interface dermatitis. The patient reports he was started on a drug a week ago. Which of the following drug could the patient have started on? A. Allopurinol B. Paracetamol C. Albuterol D. Penicillin

Penicillin Albuterol is a drug with relatively low side effects With the patient's history of Herpes Simplex Virus and targetoid lesions, it is clear he is suffering from erythema multiforme. This disease can be caused by a penicillin adverse reaction.

A 57-year-old man with abdominal pain and distention presents to the emergency room. He has vomited multiple times since yesterday. An emergency exploratory laparotomy is planned. Which of the following is recommended to decrease the chances of aspiration during general anesthesia? A. Famotidine 40 mg B. Metoclopramide 10 mg C. Nasogastric tube decompression D. Slow controlled induction of anesthesia

Nasogastric tube decompression Metoclopramide is a prokinetic drug. It increases gastric emptying and small bowel peristalsis. In patients with bowel obstruction, metoclopramide is contraindicated due to the risk of bowel perforation. Placement of a nasogastric tube in the awake patient prior to anesthesia and decompression of stomach prior to induction of anesthesia is recommended in patients with acute intestinal obstruction. Famotidine is not likely to be effective in this emergency case. The patient has distention of the abdomen and needs decompression. He is also vomiting so he may not tolerate oral famotidine. Rapid sequence induction should be done in patients at a high risk of aspiration. Slow controlled induction is recommended for patients with a cardiac problem.

A 22-year-old G1P0 at 33 weeks is presenting with preterm contractions. Her past medical history is noncontributory, and her only medication is prenatal vitamins. Her blood pressure is 134/80 mmHg, and the temperature is 37 C. Her most recent group B streptococcal swab has come back negative, and she has already received her first dose of rescue steroids. The provider also prescribes a tocolytic agent for the duration of the steroid window. Which tocolytic medication would be most appropriate for the patient? 1. Nifedipine 2. Indomethacin 3. Nitroglycerine gel 4. Intravenous alcohol

Nifedipine Tocolysis is an obstetrical procedure using medications with the focus on delaying the delivery of a fetus in women who are presenting with preterm contractions. Nifedipine and indomethacin are the two first-line tocolytic medications. Nifedipine has contraindications for cardiac disease, renal disease, and hypotension. Intravenous alcohol is no longer a medication choice for tocolysis. Nitroglycerine gel currently is not for tocolysis either. Indomethacin is contraindicated in patients above 32 weeks for fear of closure of fetal ductus arteriosis.

A 67-year-old man presents to the emergency department with a bilateral posterior headache. In triage, the patient is found to have a blood pressure of 225/105 mmHg with a heart rate of 80/min, respiratory rate of 15/min, and 100% oxygen saturation on room air. He is evaluated and diagnosed with a hypertensive emergency. Treatment with nitroglycerin is initiated. The clinician returns later, and the patient is found to be cyanotic. He complains of feeling light-headed and his heart racing. The clinician realizes the nitroglycerin drip is set at twice the expected dose. A blood gas analysis shows a PO2 of 90 mmHg and PCO2 of 36 mmHg. In addition to stopping the nitroglycerin, which of the following is the most appropriate in managing this patient? A. Administration of methylene blue B. Administration of sodium thiosulfate C. Administration of an ACE inhibitor to reduce blood pressure D. Administration of a beta-blocker to reduce heart rate

Nitroglycerin can cause cyanosis by producing methemoglobinemia and can be fatal. Methemoglobin levels can be measured. Treatment is methylene blue administration. Nitroglycerine use is contraindicated in patients with increased intracranial pressure, severe anemia, methemoglobinemia, and circulatory collapse. Nitroglycerine should not be used within 24 hours of PDE5 inhibitor use.

A 26-year-old man presents to the hospital with pain in his neck. He states the pain is worse in the morning and improves throughout the day. He states that it localizes to the upper region of his neck, and he has also noted swelling of his eye. After further evaluation, he is diagnosed with ankylosing spondylitis. Treatment options are discussed, and he agrees to a trial of sulindac. Which of the following best describes the mechanism of this agent? A. Nonselective COX inhibitor B. Angiotensin-converting enzyme inhibitor C. Opioid based analgesic D. GABA-A agonist

Nonselective COX inhibitor Sulindac is a nonselective NSAID.Nonselective irreversible inhibit COX1-and COX2 enzymes.Inhibiting COX enzymes decrease the synthesis of prostaglandins involved in pain and inflammation.ACE-I inhibitors are used in the treatment of high blood pressure and diabetic nephropathy. GABA-A agonists are used as anxiolytics.

A 30-year-old man presents to the emergency department with 2nd-degree burns to the left side of his face and body sustained in a house fire. His vital signs are within normal limits. On exam, there are extensive burns across the left cheek and periocular region and the left neck, chest, and down to his left fingers. Additionally, there are 2nd and third-degree burns across his abdomen. After irrigation, silver sulfadiazine is chosen for topical treatment of the patient's burns. In which of the following areas is it best to use a different agent in this patient? A. Chest B. Palpebral C. Distal extremities D. Abdominal

Palpebral Silver sulfadiazine cream applied topically to burns inhibits the invasion of gram-positive and gram-negative bacteria. The cream is usually applied twice daily, about 1/16th of an inch thick, over the entire burn surface. Mafenide cream may also be used for severe burns. White petrolatum is only used on superficial burns and is a protectant and moisture barrier. In the past, silver sulfadiazine was the first-line agent, but in more recent years, other topical antibiotic agents have shown the potential to be more effective.

A 16-year-old male presents with a complaint of feeling anxious when surrounded by a group of strangers. The patient reports that he fears that other people will judge him. When in a group of strangers, he can feel his heart racing and he starts sweating profusely. He denies feeling anxious when he is around family members or friends. Which of the following medications would be appropriate? A. Paroxetine B. Amitriptyline C. Olanzapine D. Gabapentin

Paroxetine is a selective serotonin reuptake inhibitor (SSRI) indicated for treating patients with SAD.Serotonin norepinephrine reuptake inhibitors (SNRIs), clonidine, and propranolol also can be used for treating patients with SAD.This patient likely is suffering from social anxiety disorder (SAD). SAD includes anxiety or marked fear in one or more social situations during which an individual may be exposed to possible scrutiny by others.Most patients with SAD benefit from treatment with a combination of psychotherapy and pharmacotherapy.

A 68-year-old male presents to the emergency department for a fever and rash. The patient reports pain on percussion of the lower back. Urinalysis shows 10 red blood cells. Complete blood count shows elevation in a certain leukocyte that are usually elevated in the blood with parasitic infections and allergic diseases. Blood cultures come back negative. The patient has a history of pneumonia a week ago caused by Streptococcus pneumoniae. What drug could have caused this patient's condition? A. Penicillin B. Acetaminophen C. Topical hydrocortisone D. Allopurinol

Penicillin The patient has a history of s. pneumoniae infection which would have been treated with penicillin. Of these drugs, only penicillin is able to cause interstitial nephritis. Acetaminophen would not be able to cause acute interstitial nephritis. Although NSAIDs could cause acute interstitial nephritis. Topical hydrocortisone would be unable to cause interstitial nephritis Allopurinol is used to treat gout, but would not be able to cause interstitial nephritis

A 61-year-old woman undergoes elective hip replacement surgery. The next day, she experiences moderate hip pain. The clinician orders IV ketorolac as needed, not to exceed a total of five days. Limiting ketorolac use to five days will most likely prevent which of the following side effects? A. Gastroesophageal reflux B. Peptic ulcer disease C. Myocardial infarction D. Renal failure

Peptic ulcer disease Ketorolac is a non-steroidal anti-inflammatory drug (NSAID). It is a non-selective COX inhibitor with a higher affinity to COX-1 than to COX-2 enzyme. Ketorolac inhibits COX-1, preventing the conversion of arachidonic acid to PGE2. PGE2 has a gastroprotective effect; therefore, inhibiting the synthesis of PGE2 can lead to peptic ulcers and gastrointestinal bleeding. According to the FDA-approved prescribing information, ketorolac should not be used for more than five days for all dosage forms. Oral ketorolac may only be used following the injectable (i.e. IV or IM) dosage form. All NSAIDs are associated with similar adverse effects on the kidneys due to the inhibition of the conversion of arachidonic acid to PGE2 and PGI2. The risk of myocardial infarction is higher with selective COX-2 inhibitors (i.e., celecoxib). NSAIDs do not cause gastroesophageal reflux.

A 44-year-old woman with chronic back pain presents to the emergency department approximately six hours after seeing her pain management specialist. She feels lightheaded. She has facial tingling, blurry vision, and a ringing in her ears. She is concerned that she is having an adverse reaction to her nerve block. With regards to toxic exposure to the likely pharmacological agent, what is the most common initial presenting symptom? A. Tinnitus B. Visual disturbances C. Perioral numbness D. Areflexia

Perioral numbness The first sign of lidocaine toxicity is oral numbness. Toxicity can result if the agent is injected errantly into the systemic circulation, into a highly vascular area, or at a dose that exceeds the maximum dose. The risk of lidocaine toxicity can be increased in patients with pseudocholinesterase deficiency. Many patients with local anesthetic toxicity present with circumoral numbness and a metallic taste.

A 65-year-old male patient is admitted to the hospital with bilateral lower limb swelling and shortness of breath. He states he has had rhinorrhea with severe myalgia and productive cough since a few days prior to this admission. On examination today he is febrile and his blood pressure is 130/90 mmHg, heart rate is 101 bpm, and oxygen saturation is 92% on room air. He has a history of osteoarthritis for which he had been using non-steroidal anti-inflammatory drugs (NSAIDs) as needed for a long time. His lab work revealed leukocytosis, with neutrophilia and an estimated glomerular filtration rate of 30 mL/min. Chest x-ray revealed a right lower lobe infiltrate. Which of the following best explains the renal dysfunction seen in this patient considering the long history of analgesic use? A. Oxidative stress secondary to NSAID use leads to free radical damage resulting in nephropathy B. Interstitial nephritis and renal papillary necrosis are rare with NSAID overuse C. Analgesic nephropathy can happen with a single drug dose but can be prevented with a short course of steroids D. Persistent inhibition of prostaglandins with analgesia overuse is likely to result in ischemic insult

Persistent inhibition of prostaglandins with analgesia overuse is likely to result in ischemic insult Inhibition of the vasodilatory effect from the prostaglandin pathway is the most recognized and accepted mechanism of hypoperfusion-related medullary ischemia, which is generally accompanied by papillary damage in the form of necrosis in the vast majority of cases. Histopathology in the vast majority of patients supports features of renal papillary necrosis and interstitial nephritis. Analgesic nephropathy occurs after years of frequent analgesic use rather than a single dose. Oral steroids have not been found to be helpful in reversing or treating this condition.

An 8-year old child presents in an unconscious state following frequent involuntary limb movements of 3-hour duration. He was promptly given intravenous lorazepam followed by phenytoin sodium with no respite. The symptoms persisted even after adding intravenous sodium valproate. A medication with which of the following mechanisms of action should be initiated in this patient? A. Inhibition of sodium channels B. Stimulation of sodium/potassium ATPase C. Stimulation of gamma-aminobutyric acid (GABA) D. Inhibition of cyclic guanosine monophosphate (cGMP)

Phenobarbital facilitates GABA-mediated neuronal inhibition and may block excitatory neurotransmitters, particularly glutamate. Phenobarbital is effective against partial and generalized tonic-clonic seizures. It is used as a second-line agent for refractory seizures not relieved by benzodiazepines, phenytoin, or valproate. Unlike benzodiazepines that also enhance GABA synapse inhibition, no antagonist exists for phenobarbital. Chronic use of phenobarbital leads to the induction of drug-metabolizing cytochrome enzyme system. Specifically, the liver 1A2, 2B6, 2C9, and 3A4/5 isozymes that will reduce the efficacy of warfarin, steroids, oral contraceptives, psychoactive drugs, and immunosuppressants.

An elderly female is taken to the emergency department by family members who are concerned about her drowsiness. The client has a history of grand mal seizures and takes an unknown antiepileptic medication. Which of the following antiepileptic medications may cause central nervous system sedation? A. Valproic acid B. Phenobarbital C. Ethosuximide D. Phenytoin

Phenobarbital is a barbiturate that is used for seizure disorders. Overdosage of phenobarbital results in central nervous system depression, respiratory failure, and hemodynamic instability. CNS depression from barbiturates can be exacerbated by alcohol use. Treatment of an overdose includes supportive care, activated charcoal, and urinary alkalinization.

A 29-year-old woman comes to the clinic to discuss contraceptive options. She recently got married and wanted to opt for a more convenient method of contraception. Past medical history is significant for seizures managed on phenytoin for the past 2 years. She has had only one seizure in the past year for which she was rushed to the emergency department and managed there. On a review of systems, she complains of occasional heartburn and seasonal allergies for which she takes cetirizine and famotidine. She, however, denies any joint pains, headaches, abnormal lumps in the breast, weight loss, breathing difficulty, or sleeping problems. She smokes half a pack of cigarettes per day and drinks an occasional glass of wine over the weekend. On examination, all systems showed no pertinent findings. Usage of combined oral contraceptive pills and their potential risks are explained in detail to the patient. The patient agrees to take combined oral contraceptives. After 2 months, she comes to the office with a positive pregnancy test. What is the most likely reason for the failure of contraception? A. Famotidine B. Cetirizine C. Smoking D. Phenytoin

Phenytoin Medications that induce the hepatic cytochrome P450 enzymes, specifically the 3A4 isoenzyme, can lead to the failure of oral contraceptives by increasing the metabolism of estrogens and progestins. Carbamazepine, phenobarbital, and phenytoin are potent inducers of the P450 system. Topiramate and oxcarbazepine are less potent inducers. When prescribing oral contraceptive pills, care must be taken to evaluate drug interactions. Phenytoin is the most common anticonvulsant reported to cause oral contraceptive failure. Cetrizine and famotidine are not P450 inducers.

A 65-year-old patient with diabetes mellitus develops orthopnea, dyspnea on exertion, and lower extremity edema for two weeks. His previous visit with the cardiologist two months ago was normal. He is on multiple medications for blood pressure and blood sugar control. Which of the following medications is the most likely cause of the patient's symptoms? A. Glipizide B. Metformin C. Pioglitazone D. Repaglinide

Pioglitazone Pioglitazone and other thiazolidinediones (TZDs) can cause or exacerbate edema and heart failure. Orthopnea, dyspnea on exertion and lower extremity edema signify heart failure. Patients on TZDs should be warned to watch for such signs and symptoms. Pioglitazone is not to be used in patients with a history of symptomatic heart failure, type one diabetes mellitus, or to treat diabetic ketoacidosis. Pioglitazone and other TZDs can cause hypoglycemia. Patients should be cautioned to watch for signs and symptoms of low blood glucose.

A 75-year-old woman with a past medical history of dementia and type 2 diabetes presents to the hospital after a nurse at the Assisted Living Facility noticed foul-smelling urine. The patient has a neurogenic bladder and requires assistance with frequent urinary catheterizations. On examination, she is febrile, has sinus tachycardia, and oriented only to self. Labs reveal leukocytosis, elevated lactate. Urinalysis reveals large leukocyte esterase with >100 white blood cells. A review of hospital records reveals that she was in the hospital two months ago for an extended-spectrum beta-lactamase (ESBL) urinary tract infection (UTI). The patient was started on meropenem two days ago. However, her fever persists and she is worsening. What is the most likely mechanism of resistance of the bacterium to meropenem? A. Efflux pumps B. Reduced permeability of lipopolysaccharide layer C. Insertion sequences via transposition D. Plasmid-mediated transmission

Plasmid-mediated transmission Carbapenems have been shown to be the most potent treatments for multidrug-resistant Enterobacter infections. Meropenem and Imipenem have been shown to be effective against E. cloacae and E. aerogenes, two of the most clinically relevant enterobacter species. The main mechanism in which Enterobacter becomes resistant to carbapenems involves multiple enzymes that are plasmid-mediated, leading to accessible horizontal transfer and development of widespread resistance. Prolonged stay at health care facilities and persistence of invasive medical devices is one of the leading risk factors for antibiotic-resistant bacteria. While the presence of a lipopolysaccharide (LPS) capsule can aid Enterobacter in avoiding opsonophagocytosis, it is not the main mechanism of Enterobacter resistance to carbapenems. While the upregulation of efflux pumps can be seen in some carbapenem-resistant Enterobacter, it is not the main mechanism of resistance.

A 30-year-old male is found on the street and brought into the emergency department for altered mental status. A computed tomography scan of the head is negative. The patient is found to have urine toxicology positive for opioids. He is treated and regains consciousness. He is later treated with naltrexone. What is the role of naltrexone in treating this patient's condition? A. Treat withdrawal symptoms B. Prevent respiratory failure C. Prevent relapse D. Remove the physical dependence

Prevent relapse This patient has acute opioid toxicity. Naloxone is used for acute treatment. Once stabilized, the patient should be encouraged to enter treatment. Naltrexone is an opioid receptor antagonist. It is used to treat alcohol and opioid dependence. It can be given either daily in oral form or monthly as an injection. Naltrexone is not recommended in patients with chronic pain. The usefulness of naltrexone for opioid dependence is limited because of low retention and poor compliance. Some claim witnessed daily ingestion by a significant other improves compliance and effectiveness. The monthly injection also seems to improve compliance. Naltrexone temporarily reduces substance intake and cravings because the patient knows that opioid use would be pointless until the naltrexone is gone.

A female visits a contraception clinic for counseling. The client is interested in progesterone-only pills. What is the primary mechanism of action of this mode of contraception? A. Thickening of cervical mucus B. Spermicidal action C. Prevention of implantation D. Prevention of ovulation

Prevention of ovulation The main mechanism of action of progesterone pills is by the prevention of ovulation. Progestogen negative feedback works at the hypothalamus to decreases the pulse frequency of the gonadotropin-releasing hormone. This will decrease the secretion of follicle-stimulating hormone (FSH) and luteinizing hormone (LH). If the follicle isn't developing, then there is no increase in the estradiol levels (the follicle makes estradiol). The progestogen negative feedback and lack of estrogen positive feedback on LH secretion stop the mid-cycle LH surge. With no follicle developed and no LH surge to release the follicle, there is the prevention of ovulation.

An 80-year-old man is hospitalized for an intra-abdominal infection. He is receiving intravenous ciprofloxacin and metronidazole and ondansetron as needed for nausea. On day three of admission, he becomes restless, agitated, and aggressive towards the hospital staff. The attending clinician decides to administer a dopamine antagonist to manage his agitation. Which of the following complications is most likely to occur following administration of this drug? A. QT interval prolongation B. Disulfiram-like reaction C. Arthralgia D. Vomiting

QT interval prolongation Haloperidol is a dopamine receptor blocker that may be used for treating acute agitation. It can prolong cardiac conduction (prolonged QT interval).This patient is at risk of torsade de pointes because he is receiving two other medications (ciprofloxacin and ondansetron) that can cause QT prolongation. The risk is higher with the IV formulation compared to the oral formulation.Haloperidol should not be administered if a prolonged QT exists already. Monitor ECG before administration and continuously.All typical antipsychotics increase the risk of QT prolongation.

A 32-year-old male is referred for evaluation of treatment for opioid dependence. The patient has been part of various rehabilitation programs but has had two episodes of relapse in the last year. He was registered in a group of patients who were trialed with methadone. He failed the trial and dropped out. The clinician starts a trial of buprenorphine to treat his opioid dependence. Which of the following potential side effects of methadone is most likely to occur with this therapy as well? A. Pruritis B. Increase the seizure threshold C. QT interval prolongation D. Diarrhea

QT interval prolongation The combination of buprenorphine and naloxone is used to treat patients with opioid dependence. It is used for induction in patients dependent on short-acting opioids. Buprenorphine monotherapy is used for induction in patients dependent on long-acting opioids. It also can be used as a maintenance drug. A possible side effect is QT prolongation. This is significantly less than with methadone. As with methadone, providers must be cognizant before prescribing to patients with cardiac rhythm issues. It is administered as a sublingual film or tablet or a buccal film. Buprenorphine is a high-affinity drug with partial opioid agonist activity, and naloxone is a potent opioid antagonist. The combination drug can precipitate withdrawal in opioid-dependent patients. Common adverse effects include constipation, headache, pain, and insomnia.

A 66-year-old male with a recent diagnosis and treatment for a urinary tract infection presents with acute pain over his Achilles tendon. He denies any trauma or change in activity level. Which of the following is the most likely cause of his pain? A. Advanced age predisposing him to tendinosis B. Recent fluoroquinolone treatment for the urinary tract infection C. Autoimmune disease D. Acute non-traumatic rupture

Recent fluoroquinolone treatment for the urinary tract infection The FDA recommends that at the first sign of tendon pain, swelling, or inflammation, patients should stop taking the fluoroquinolone, avoid exercise and use of the affected area, and promptly contact their provider for tendon evaluation and transition to a non-fluoroquinolone antibiotic. The median number of days before onset is eight. It is important to obtain a medical history when evaluating tendinopathy. Fluoroquinolone-associated tendinopathy most often affects the Achilles tendon.

A 65-year-old man with metastatic prostate cancer to bone presents with bone pain. He has had no pain relief with aspirin. The clinician decides to choose a drug from the next level of the analgesic ladder. Which side effect of the next most appropriate drug is independent of tolerance development? A. Respiratory depression B. Miosis C. Pruritis D. Addiction

Respiratory depression According to the World Health Organization's (WHO) pain ladder for cancer pain relief in adults, prompt oral administration of medication is warranted. Nonopioids, such as aspirin and acetaminophen, should be given first. If necessary, mild opioids, such as codeine, should be given next. If mild opioids are unable to bring adequate pain relief, then strong opioids, such as morphine, should be given until the patient is free of pain. Tolerance does not develop to miosis and constipation. The WHO also recommends adjuvant medications "to calm fears and anxiety" as needed.

A patient is going on a sea cruise but has previously had severe motion sickness. What is the best prophylactic agent to prevent nausea and sea sickness for this individual? A. Cimetidine B. Scopolamine C. Meclizine D. Chlorpromazine

Scopolamine Scopolamine is an anti-muscarinic agent, which has proven to be effective for motion sickness and nausea. Scopolamine is used as a patch that is placed behind the ear. Scopolamine does not cause excess sedation. Scopolamine can also induce pupillary dilatation and cycloplegia; thus it can be used to treat inflammatory eye disorders like iritis and uveitis.

A 34-year-old woman G1P0 at 34 weeks of gestation comes for her regular appointment to the clinic. Her pregnancy has been complicated by a diagnosis of preeclampsia starting in the second trimester. During this visit, she states she noticed increased nausea and vomiting. A healthcare provider prescribes her a scopolamine patch. What is the most feared complication of this treatment? A. Dry mucous membranes and blurry vision B. Severe cholinergic syndrome C. Severe anticholinergic syndrome D. Eclamptic seizures

Scopolamine can cause eclamptic seizures in pre eclamptic patients. Preeaclampsia is a result of placental vasospasms. The use of anticholinergic agents may leave the sympathetic system unopposed. Worsening vasospasms may worsen preeclampsia and results in progression to eclampsia with seizures.

A 65-year-old female presents after her spouse found her unconscious lying face down on the floor. She has a 30-year history of generalized anxiety disorder and panic attacks. Her respiratory rate is 6/min, blood pressure 80/70 mmHg, the temperature is 98.0 F (36.7 C), and heart rate 60 beats/min. The patient is unable to cooperate with a physical examination. Intravenous medication is administered, which improves the patient's vital signs within ten minutes. Her respirations are now 13/min, and her blood pressure is 110/80 mmHg, pulse 80/min, and temperature 98.0 F (36.7 C). Which of the following is a potential adverse effect associated with the rapid administration of the medication this patient most likely received? A. Seizure B. Nausea C. Myalgia D. Bradypnea

Seizure Flumazenil is a benzodiazepine antagonist. Flumazenil works immediately when administered intravenously. It has an initial half life of 7-15 minutes. Flumazenil administration can provoke a seizure, especially in a patient with a chronic history of benzodiazepine use. Flumazenil injection is indicated for a complete or partial reversal of the sedative effects of benzodiazepines in conscious sedation and general anesthesia in the adult and pediatric populations. Flumazenil speeds the recovery from sedation following minor surgical procedures and shortened the postoperation monitoring period for minor surgery, resulting in earlier patient discharge. Flumazenil is also indicated for the management and treatment of benzodiazepine overdose in adults. It is useful in reversing coma due to benzodiazepine overdose. Flumazenil is more effective in reversing sedation or coma in patients with benzodiazepine intoxication rather than in patients with multiple drug overdoses. Serious adverse effects of flumazenil include neurologic effects, seizures, and arrhythmias.

A 35-year-old man presents to the clinic for follow-up. He has a history of partial epilepsy and has been taking oral carbamazepine and phenytoin. He reports poor control of his symptoms. His baseline investigations are normal. A novel antiepileptic agent of the sulfonamide group, which exerts action on T-type calcium channels, is added to his regimen. The patient presents to the emergency department a few weeks later with hyperventilation, fatigue, and anorexia. Physical examination is unremarkable. Which of the following lab parameters is most consistent with this patient's current presentation? A. Serum glucose of 350 mg/dL B. Serum creatinine of 3.5 mg/dL C. Serum bicarbonate of 18 mEq/L D. Serum sodium of 125 mEq/L

Serum bicarbonate of 18 mEq/L This patient has a history of partial epilepsy that has been difficult to control. The drug that the patient is started on is zonisamide, a sulfonamide-based antiepileptic agent used as an adjunct for partial seizure. Zonisamide acts by blocking T-type calcium channels and sodium channels. It also functions as a carbonic anhydrase inhibitor and can cause metabolic acidosis. This side effect has an FDA warning. The reduction in bicarbonate was noted to be mild, about 2 mEq/L in most patients; however, severe reduction of up to 10 mEq/L has also been reported. The risk is higher at elevated drug doses but can occur at lower doses. Hyperglycemia, hyponatremia, and renal failure are not seen in an appreciable frequency with zonisamide use.

A female is convinced she has hypoglycemia. She complains of shakiness, nausea, and irritability 3 hours after meals. In the morning she feels fine, even though she has not eaten since 7 PM the previous evening. Which of the following is true about this patient? A. She has fasting hypoglycemia B. She symptoms are secondary to anxiety C. Her symptoms are secondary to an insulinoma D. She has postprandial hypoglycemia

She has postprandial hypoglycemia If the patient had fasting hypoglycemia she would have symptoms in the morning. A patient with an insulinoma would be hypoglycemic in the morning. Postprandial hypoglycemia tends to occur 3-4 hours after a meal, especially after consuming meals with a high content of simple carbohydrates. The postprandial symptoms are classically due to a reactive cause. It is important to note that some patients with an insulinoma can also present with postprandial hyperglycemia.

A 17-year-old female arrives at the emergency department via ambulance. On exam, she is agitated and confused. Her blood pressure is 160/110 mmHg, heart rate 140 beats/min, and temperature 39C. She is diaphoretic, tremulous, and hyper reflexive but otherwise has a non-focal neurological exam. She is unable to give a detailed history. She has with her a bottle of phenelzine. Her friends report that she "wanted to celebrate her eighteenth birthday" with "something new." What other drugs may she have taken to cause this presentation? A. MDMA (ecstasy) B. Marijuana C. Phencyclidine (PCP) D. Lorazepam

She is exhibiting neuromuscular effects, autonomic effects, and mental status changes associated with serotonin syndrome. She was taking phenelzine, a monoamine oxidase inhibitor (MAOI). The addition of methylenedioxymethamphetamine (MDMA), also known as ecstasy, or cocaine may precipitate serotonin syndrome. Serotonin syndrome occurs following the use of serotonergic drugs. The degree of symptoms can range from mild to severe. Symptoms include hyperthermia, agitation, increased reflexes, tremors, sweating, dilated pupils, and diarrhea. Temperature can increase to greater than 41.1C. Serotonin syndrome typically is caused by a combination of an SSRI with other serotonergic substances such as serotonin-norepinephrine reuptake inhibitors, MAOIs, tricyclic antidepressants, amphetamines, buspirone, triptans, St. John's wort, ecstasy, or cocaine. It is a predictable consequence of excess serotonin. The onset of symptoms occurs within a day of serotonin toxicity. Treatment consists of stopping the serotonin. In those who are agitated, benzodiazepines and a serotonin antagonist such as cyproheptadine may be used. In those with hyperthermia, active cooling measures may be needed. With treatment, the risk of death is less than 1%.

A 45-year-old female with a history of seizures that was well controlled with valproic acid presents to the office. The patient developed type 2 diabetes mellitus after being seizure-free with no changes occurring in her valproic acid over that time period. The practitioner started an oral medication to help control postprandial sugars levels. The patient started to experience severe flatulence and diarrhea. Three months after starting the medication, she was found to have elevated serum transaminases and had 2 seizures during that time period after being seizure-free for the past 5 years. What is the likely mechanism of action of the drug prescribed by the practitioner? A. Increases insulin release from pancreatic beta-cells B. Decreases glucose production by the liver C. Increases insulin sensitivity in peripheral tissues D. Slows the digestion of ingested carbohydrates

Slows the digestion of ingested carbohydrates Acarbose inhibits pancreatic alpha-amylase and intestinal alpha-glucoside hydrolase in the brush borders of the small intestines which result in the slowing of digestion of complex carbohydrates which results in decreased glucose absorption. Diarrhea and flatulence is a commonly observed side effect. Acarbose usually does not lead to hypoglycemia when administered as monotherapy. It is important to treat hypoglycemia with oral glucose (dextrose) and not sucrose (table sugar) since the hydrolysis of sucrose and fructose is inhibited by acarbose. Acarbose may decrease the bioavailability of digoxin and valproic acid Acarbose helps lower postprandial rise of blood sugar levels.

A 45-year-old woman with HIV presents with complaints of fever, dysuria, flank pain, and confusion. Her temperature is 102.9 F, blood pressure 85/50 mmHg, pulse rate 125/min, and respiratory rate 28/min. Laboratory analysis reveals an anion gap of 20 mEq/L. Which of the following should be administered to treat this client's acid-base disturbance? A. Furosemide B. Sodium bicarbonate C. Hypertonic saline D. Normal saline

Sodium bicarbonate This client has developed a high anion gap metabolic acidosis due to sepsis. High anion gap metabolic acidosis is one of the most common metabolic derangements seen in critical care patients.Primary treatment of high anion gap metabolic acidosis requires correction of the underlying cause. In mild to moderate acidosis, treatment can be limited to supportive measures including intravenous fluids and respiratory support.Alkalization therapy should be reserved for those individuals with severe acidosis. The most common alkalizing agent is sodium bicarbonate.The main therapeutic effect of sodium bicarbonate administration is in increasing plasma bicarbonate levels. Bicarbonate is known to buffer excess hydrogen ion concentration, thereby raising solution pH to counter clinical manifestations of acidosis.

A 66-year-old man with a history of poorly controlled hypertension presents to the emergency department with sudden worsening of shortness of breath over the past 6 hours. He states that he ran out of his blood pressure medications a few days ago and he couldn't get a ride to the pharmacy. His vitals on arrival are remarkable for a heart rate of 82 and a blood pressure of 205/110. A drug is started by the nurse via an IV infusion whose effect involves balanced arterial and venous dilation. Which of the following drugs did the nurse give? A. Hydralazine B. Minoxidil C. Nitroglycerin D. Sodium nitroprusside

Sodium nitroprusside Sodium nitroprusside is a drug used to relax both arteries and veins. The drug is administered intravenously. Hydralazine is a direct vasodilator which dilates arterioles with little effect on veins. Nitroprusside acts by releasing nitric oxide. Nitroprusside slowly breaks down to release cyanide when exposed to light. The duration of treatment should not be for more than 72 hours and thiocyanate concentrations should be monitored. Sodium nitroprusside is also used in ketone strips.

A 54-year-old woman with a past medical history of hypertension, hyperlipidemia, tobacco use disorder, and alcohol use disorder is hospitalized for community-acquired pneumonia. On morning rounds, she says she "could not sleep at all" through the night. She also complains of nausea, palpitations, and anxiety. Her morning vital signs show her blood pressure to be 187/100 mmHg, heart rate 120 bpm, respirations 18/min, oxygen saturation 95% on room air, and an oral temperature 98.6 F (37 C). Her cardiac exam demonstrates regular rhythm without murmurs, rubs, or gallops, and the pulmonary exam is notable for stable crackles in the right lower lobe. Her calculated CIWA score is 16. What is the most appropriate next step in management? A. Start IV labetalol to prevent hypertensive emergency B. Start IV ethanol infusion to prevent delirium tremens C. Start oral folate to prevent Wernicke-Korsakoff syndrome D. Start IV phenobarbital to prevent alcohol withdrawal seizures

Start IV phenobarbital to prevent alcohol withdrawal seizures Alcohol withdrawal needs to be managed very aggressively early on, as it can progress to alcohol withdrawal seizures and delirium tremens. Phenobarbital and benzodiazepines are used to prevent seizures in those patients who have signs of active alcohol withdrawal. It is best treated with IV medications rather than oral medications. The Clinical Institute Withdrawal Assessment for Alcohol, revised (CIWA-Ar), can be used to identify patients who are at risk and to monitor treatment. Delirium tremens is the most severe form of alcohol withdrawal. Patients will present with confusion and signs of autonomic hyperactivity. It is fatal in nearly 15% of patients without treatment and 1% in those who do receive treatment. Major alcohol withdrawal presents with visual and auditory hallucinations, diaphoresis, and hypertension. Minor alcohol withdrawal presents with anxiety, nausea, vomiting, insomnia, and tremor. Intravenous ethanol infusion is not recommended for prophylaxis or treatment of alcohol withdrawal, including delirium tremens. Repleting thiamine (B1) can reduce the risk of Wernicke encephalopathy, an acute neurological condition characterized by a clinical triad of ophthalmoparesis with nystagmus, ataxia, and confusion.

A 70-year-old woman, with a recent diagnosis of lung cancer, presents to the clinic with mild body pain and malaise. She also complains of nausea, vomiting, and loss of appetite since she began her chemotherapy. She is currently taking ondansetron and dronabinol, along with the prescribed chemotherapy. Her pulse rate is 78/min, the temperature 98.6 F, blood pressure 140/70 mmHg, and respiratory rate 15/min. She is prescribed a cyclooxygenase inhibitor for the pain. This drug most likely occupies which step of the WHO analgesic ladder? A. Step 1 B. Step 2 C. Step 3 D. Step 4

Step 1 Step one of the WHO analgesic ladder describes a non-opioid. Step two of the WHO analgesic ladder involves a weak opioid. Step three involves the use of a strong opioid. The pain ladder or analgesic ladder was created by the WHO as a guide for medical professionals to reference when using drugs to manage all types of pain. The general principle is to begin with the first step, then climb the ladder if pain persists. The pain medications range from common, over-the-counter drugs at the lowest rung of the ladder to strong opioids. The WHO guidelines recommend rapid oral administration of pain-relieving drugs when pain occurs, starting with non-opioid drugs such as paracetamol (acetaminophen) or aspirin, with or without non-steroidal anti-inflammatory drugs or COX-2 inhibitors. If pain relief is not achieved, more aggressive treatment with a weak opioid, such as codeine, tramadol, or dihydrocodeine is added to the non-opioid regimen. If this is insufficient, a weak opioid is replaced by a strong opioid, such as morphine, diamorphine, buprenorphine, fentanyl, oxymorphone, oxycodone, or hydromorphone, and continuing the non-opioid therapy while escalating the opioid dose until the patient is pain-free or has reached the maximum possible relief without intolerable side effects. If the initial presentation is severe pain, this process should be skipped, and a strong opioid should be initiated immediately in combination with a non-opioid analgesic. The guidelines indicate that medications should be given at regular intervals so that continuous pain relief occurs, and dosing by actual relief of pain occurs rather than fixed dosing guidelines. It recognizes that breakthrough pain may occur, and directs how immediate rescue doses should be provided. The usefulness of the second step, a weak opioid, is debated due to their higher toxicity and low efficacy, and many argue that a weak opioid could be replaced by smaller doses of a strong opioid. Not all pain yields to classic analgesics and drugs such as steroids or bisphosphonates also may be employed concurrently with analgesics at any stage. Tricyclic antidepressants, anticonvulsants, or class I antiarrhythmics are drugs of choice for neuropathic pain. Up to 90% of patients with cancer use such adjuvants.

A 52-year-old woman, after being diagnosed with coronary artery disease, is started on a medication. The medication acts on the cyclo-oxygenase pathway and is known to reduce the risk of cardiovascular disease. Where is the majority of this medicine absorbed? A. Stomach B. Duodenum C. Ileum D. Colon

Stomach This client is started on aspirin. This medicine inhibits the cyclo-oxygenase pathway. Aspirin is rapidly absorbed through the gastric mucosa. When consumed as a liquid preparation, it is rapidly absorbed as opposed to tablets. Aspirin is a weak acid. In the stomach, due to acidic pH, the drug molecule has no net charge and is rapidly absorbed by passive diffusion. Although the aspirin is majorly absorbed from the stomach but at the same time, it increases the risk for gastritis.

A 70-year-old male presents to the clinic for a follow-up appointment for type 2 diabetes. The patient has brought a blood sugar log that shows his blood sugars have occasionally been as low as 40mg/dl when he awakes in the morning. He adds that at this blood sugar level, he prefers to chew a candy. His HbA1c two months ago was 7.2, and his current medication regimen for diabetes includes metformin, glyburide, liraglutide, and dapagliflozin. What is the most appropriate adjustment in his current diabetic regimen? A. Continue current medication regimen B. Stop glyburide C. Stop liraglutide D. Stop dapagliflozin

Stop glyburide Sulfonylureas have a known side effect of causing hypoglycemic episodes. Examples of sulfonylureas include glipizide, glimepiride, and glyburide. Particularly in the elderly population, hypoglycemia may go unrecognized, and these patients may experience difficulty communicating their symptoms to others. When compared to other sulfonylureas in the second generation, glyburide has been shown to possess a higher hypoglycemic risk.

A 78-year-old female with hypertension and diabetes mellitus is new to the practice. She is on lisinopril 40 mg a day and metformin extended-release 1 gram a day. Her blood pressure is well controlled, and the exam is unremarkable. Laboratories show a glucose of 190 mg/dL, hemoglobin A1c 6.5%, potassium 4.6 mEq/L, creatinine 1.8 mg/dL, and 1+ protein in the urine. The estimated glomerular filtration rate is 28. A review of her records shows an increase in her creatinine compared to a year ago. What is the appropriate management? A. Continue the same medications B. Stop metformin and start another oral hypoglycemic medication C. Decrease the dose of lisinopril D. Discontinue lisinopril

Stop metformin and start another oral hypoglycemic medication This patient has renal insufficiency, making continuing metformin inappropriate because of the increased risk of lactic acidosis. Diabetic nephropathy is best treated with ACE inhibitors, so lisinopril should be continued with monitoring of creatinine and potassium. Her blood pressure is well controlled; therefore, no change in dose is needed. The hemoglobin A1c is acceptable but will need to be rechecked 3 months after initiating a new oral hypoglycemic. Metformin is contraindicated in severe renal dysfunction with an eGFR <30 mL/min/1.73 m².

In a patient with tuberculosis, which of the following aminoglycosides is frequently used as a secondary drug? A. Kanamycin B. Neomycin C. Gentamycin D. Streptomycin

Streptomycin is a first generation aminoglycoside that was first used to treat tuberculosis. Today, it is rarely used because safer drugs are available. Streptomycin acts on the 30S ribosomes and prevents protein synthesis. At lower concentrations, streptomycin is only bacteriostatic. Besides treating tuberculosis, streptomycin is used to treat plague, infective endocarditis, and various gram-negative organisms. Streptomycin cannot be administered orally and has to be given via intravenous injection or intramuscularly.

A 33-year-old male comes to the office for follow up after recently beginning therapy for pulmonary tuberculosis. His regimen includes isoniazid, ethambutol, pyrazinamide, and streptomycin. He is currently feeling well and is having no side effects. Upon returning to the clinic for a second follow up appointment, he complains of progressive difficulty hearing and having several episodes of vertigo. Which of his medications is likely responsible for these side effects? A. Isoniazid B. Ethambutol C. Pyrazinamide D. Streptomycin

Streptomycin is an antibiotic used in combination with other drugs to treat tuberculosis (TB). TB is a chronic bacterial infection. Aminoglycosides can damage the eighth cranial nerve, also known as the acoustic nerve. Hearing loss or alterations in noise perception are potential complications and reported immediately. Ototoxic effects of aminoglycosides, such as streptomycin, may find themselves potentiated by the co-administration of furosemide, mannitol, and ethacrynic acid. Renal function should be monitored in these patients and can play a role in toxicity potential.

A 17-year-old male is brought in by his friends because he was "hallucinating about a demon ordering him to slay them to receive the graces of God." The patient has no history of a psychiatric disorder. His friends report having a graduation party at home where they ground amphetamine-based attention-deficit/hyperactivity disorder pills and snorting the powder. Which of the following will most likely be present in this patient? A. Hypertension and constricted pupils B. Bradypnea and dilated pupils C. Tachypnea and constricted pupils D. Tachycardia and dilated pupils

Tachycardia and dilated pupils This patient is suffering from amphetamine-based stimulant toxicity. Amphetamines increase the activity of the neurotransmitters dopamine and norepinephrine. They also induce the release of epinephrine, serotonin, and histamine. Increased dopamine levels in the central nervous system cause psychosis, euphoria, and movement disorders. Tachycardia is due to the sympathomimetic effects of amphetamines. Amphetamine toxicity causes dilated pupils. Amphetamine toxicity also causes tachypnea.

A female was out in the woods during the late summer and soon developed a rash and arthralgias. She was immediately started on a drug. Three weeks later, a biopsy of the liver revealed microvesicular fatty change. What was the most likely drug? A. Tetracycline B. Gentamicin C. Trimethoprim-sulfamethoxazole D. Alcohol

Tetracycline Microvesicular fatty liver changes occur in acute fatty liver of pregnancy, drug toxicities (salicylate, valproic acid, tetracycline), Reye syndrome, or inborn metabolic defects (of the urea cycle enzymes or FFA oxidation). It appears that the patient had developed Lyme disease and was treated with a tetracycline. In fatty liver, there is an accumulation of fat and triglycerides in the liver cells. Most patients have no symptoms but if the condition is allowed to progress it can lead to cirrhosis. With tetracycline, if the drug is discontinued, the condition resolves.

When is the best time to apply transdermal scopolamine for nausea? A. Preoperatively B. Intraoperatively C. Postoperatively D. Anytime

The scopolamine patch is best to apply before the onset of nausea. Scopolamine is widely used to treat motion sickness. It is an antimuscarinic agent and thus not recommended in people with glaucoma. Side effects include dry mouth, tachycardia, pruritus, and constipation. It is available as a patch that can be placed behind the ear and it works within 30 minutes. Alcoholic drinks in South America are often spiked with this agent when committing a robbery on unsuspecting tourists.

A test subject in a phase 1 clinical trial receives an intravenous injection of an experimental drug that is structurally similar to a primary neurotransmitter in the sympathetic nervous system. Which of the following will most likely occur after receiving the experimental drug? A. Decreased cardiac output B. Increased cardiac contractility C. Decreased cardiac stroke volume D. Increased cardiomyocyte sodium

The experimental drug is structurally similar to norepinephrine. Norepinephrine acts on beta-1 adrenergic receptors within the heart. Targeted activation of the beta-1 receptor in the heart increases sinoatrial (SA) nodal, atrioventricular (AV) nodal, and ventricular muscular firing, thus increasing heart rate and contractility. With these two increases, the stroke volume and cardiac output will also increase. The Gs subunit of the beta-1 adrenoreceptor upon activation upregulates adenylyl cyclase which converts ATP to cAMP. With the increased presence of cAMP, cAMP-dependent protein kinase A (PKA) phosphorylates calcium channels, thus increasing cellular calcium influx. Increased concentrations of intracellular calcium increase inotropy in the heart through calcium exchange in the sarcoplasmic reticulum. PKA also phosphorylates myosin light chains which lead to contractility in smooth muscle cells. Many drugs derive their pharmacology from their interaction with the sympathetic and parasympathetic nervous systems. For example, beta-adrenergic receptor blockers exert their pharmacologic effect by inhibiting natural ligands, e.g., norepinephrine and epinephrine, from binding to specific beta-adrenergic receptors.

A 15-year-old male was brought to the emergency department of a rural hospital with chief complain of moderate to severe muscular weakness. The patient also has weak respiratory effort and copious oral and nasal secretions. The father is unsure whether this is an allergic reaction to the insecticides that was sprayed over the farm last night by a local contractor. The patient's father also has a mild headache and is complaining of blurry vision with mild abdominal cramps. The physical examination is significant for bilateral rhonchi and hyperactive bowel sound. Pupils are 2 to 3 millimeters bilaterally and sluggishly reactive to light. Which of the following diagnosis is the most probable in this patient? A. Allergic reaction to insecticides B. Exacerbation of asthma C. Organophosphate poisoning D. Tick paralysis

The patient most likely has organophosphate poisoning. The patient is exhibiting signs of or cholinergic crisis which is from organophosphate poisoning. A useful mnemonic is SLUDGEM (S - salivation, L - lacrimation, U - urinary frequency, D - diarrhea, G - gastrointestinal cramping and pain, E - emesis, and M - miosis). Acute or chronic exposure to pesticides and insecticides containing organophosphates can trigger a cholinergic crisis. Apart from the muscarinic and nicotinic effects seen in cholinergic crisis, patients might also exhibit neurological symptoms like headache, dizziness, tremor, and paresthesia.

A 45-year-old alcoholic man is admitted to the intensive care unit following a head injury. Five milligrams of diazepam is administered intravenously to him every 2 hours, as needed, according to the instructions given. After administering a dose, you observe that the patient's respiratory rate has changed from 20/min to 6/min. Which drug should be readily available to treat this complication? A. Fentanyl B. Fluorouracil C. Naloxone D. Flumazenil

The patient should be given flumazenil. Such a marked drop in the respiratory rate is most likely due to the effects of diazepam. Flumazenil is a competitive benzodiazepine receptor antagonist and is used as an antidote for benzodiazepine toxicity. The adult dose is 0.2 mg IV over 15 seconds. If there is no response after 45 seconds, then administer 0.2 mg again over 1 minute. This may be repeated at 1-minute intervals and should not exceed 4 doses (1 mg, including the initial dose).

A 62-year-old male is brought to the emergency department by his wife when she found him lying in bed with an empty bottle of phenobarbital in the bathroom. The client has severe respiratory depression at present with a respiratory rate of 4 bpm. What would the nurse comprehend as the initial management of severe respiratory depression in the client? A. Hemodialysis B. Naloxone C. Flumazenil D. Artificial ventilation

There is no antidote for barbiturate overdose, flumazenil only antagonizes benzodiazepines. If respiratory depression occurs, endotracheal intubation and mechanical ventilatory support are appropriate. Barbiturates have a long half-life. Phenobarbital has an elimination half-life of 70 to 140 hours, and since 4 to 5 half-lives are required to eliminate a drug it could take a week or longer to recover. In extremely high serum concentrations hemodialysis will hasten clearance and short coma. Barbiturates have very few applications in clinical medicine, but still are used for epilepsy, especially if someone had been started on this years ago and were stable, such as an elderly patient presented here. Supportive therapy with hydration is the best method of treating barbiturate overdose. Continuous monitoring of the client's airway is essential. Barbiturates also lower body temperature, so the body must be warmed. Aggressive IV hydration is required to counter low blood pressure. Off the barbiturates, only phenobarbital may have it's renally excretion enhanced by alkalinization with sodium bicarbonate, but initial management is ABC, and airway management is critical.

A 75-year-old man is being treated for malignant hypertension with an IV infusion of a vasodilator drug. Aside from chronic hypertension, his past medical history is notable for type 2 diabetes mellitus, chronic kidney disease stage III, and hypercholesterolemia. On the second day of admission, he complains of ringing in his ears and abdominal pain. The nurse also reported that he became increasingly agitated and disoriented overnight. His blood pressure this morning is 145/98 mmHg, and the remaining vital signs are all within normal limits, with an oxygen saturation currently at 95%. An increased serum concentration of which of the following is most likely to be the cause of this patient's symptoms? A. Cyanide B. Methemoglobin C. Thiocyanate D. Cyanmethemoglobin

Thiocyanate Sodium nitroprusside combines with oxyhemoglobin to ultimately produce five cyanide ions, an active nitroso group (NO), and methemoglobin. The most important route of cyanide detoxification involves the enzyme rhodanese, which converts cyanide to water-soluble thiocyanate, facilitating excretion in urine. Thiocyanate toxicity is a rare toxicity syndrome associated with nitroprusside; it is most commonly seen in the setting of prolonged infusions (>72 hours) and/or renal dysfunction due to slow clearance by the kidney. Thiocyanate levels should be monitored in patients with renal dysfunction who are receiving nitroprusside infusions. Unlike thiocyanate toxicity, renal dysfunction is not an established risk factor for cyanide toxicity. Cyanide levels are also not typically available expeditiously enough to be clinically useful; instead, clinicians must use surrogates such as pH and mixed venous O2 saturation. The symptoms of thiocyanate toxicity manifest as tinnitus, nausea/vomiting, abdominal pain, weakness, lethargy, agitation, or disorientation, and ultimately progress to seizure and coma if untreated. In renal failure patients, dialysis is often required to facilitate thiocyanate clearance in situations of toxicity.

A 52-year-old male is found by a friend. The friend called 911 because he could not wake him up. He told the 911 operator that his friend was still breathing but barely. The patient is found to be very lethargic, bradycardic, apneic, hypotensive, and has pinpoint pupils. His point of care glucose is 72 mg/dL. What is the best treatment for him? A. Flumazenil B. Folic acid C. Naloxone D. Glucose

This case is a classic presentation of opioid overdose causing respiratory depression, hypotension, and pinpoint pupils. The best treatment is naloxone.Airway protection must be maintained. Assisted ventilation will be needed with a bag valve mask until naloxone reverses the respiratory depression. Adequate ventilation should be provided before naloxone is given. The half-life of naloxone in adults is approximately 30 to 80 minutes with an average of 60 minutes. It is about 3 hours in children.Naloxone continuous infusions may be required in major overdoses. When given intravenously, naloxone works in about 2 minutes. It works within 5 minutes when administered intramuscularly. In adults, the dose is 0.4 mg to 2 mg and may be repeated every 2 to 3 minutes up to a max dose of 10 mg or until a full reversal is achieved. The pediatric dose is 0.1 mg/kg for children under 5 years of age or 20 kg. If over 20 kg or age five years, a dose of 2 mg is administered intravenously for rapid reversal of respiratory depression or arrest.After naloxone administration, patients often begin vomiting violently and become combative. Staff safety and airway protection of the patient is a priority.

A 55-year-old man presents to the clinic for a regular followup. He was recently diagnosed with diabetes and was started on a few medications. One of the medications helps to decrease the risk of coronary artery disease. What is the mechanism of action of this drug? A. Increase cyclic GMP B. Increase thromboxane formation C. Inhibit cyclooxygenase D. Inhibit adenosine diphosphate-induced aggregation

This client is started on aspirin to reduce the risk of coronary artery disease. Aspirin is a cyclooxygenase-1 (COX-1) inhibitor. It is a modifier of the enzymatic activity of cyclooxygenase-2 (COX-2). Due to the blocking of the COX pathway, the arachidonic acids are shuttled into the lipoxygenase pathway. The production of anti-inflammatory lipoxins is a result of the modification of prostaglandin-endoperoxide synthase (PTGS2), also called COX-2, that results in the production of lipoxins, most of which are anti-inflammatory. Unlike other NSAIDs (ibuprofen/naproxen), which bind reversibly to this enzyme, aspirin binding is irreversible. It also blocks thromboxane A2 on platelets in an irreversible fashion preventing platelet aggregation. Aspirin produces both analgesic and anti-inflammatory effects. Almost 90% of COX inhibition can be achieved with administration of 160 to 325 mg of aspirin. These effects last for about 7 to 10 days, which usually correspond with the lifespan of a platelet.

A 69-year-old comes into the hospital with chest pain and is started on a tissue plasminogen activator (tPA). The patient does well and is discharged after two days. What is tPA? A. Coronary vasodilator B. Fibrinolytic agent C. Digoxin analogue D. Bronchodilator

Tissue plasminogen activator is a thrombolytic agent that dissolves blood clots in the vessels of the heart or other organs.It should be used as soon as possible after a heart attack.It is also indicated for pulmonary embolism and ischemic stroke.The therapy can be used in children in the case of parapneumonic effusion and empyema.

An adult male admitted in the inpatient hematology unit repeatedly develops a fever following blood transfusions with leukoreduced red blood cells (RBCs). What would the nurse comprehend as the most appropriate measure in the client in this scenario? A. Administer freshly frozen RBCs B. Release 41 day old RBC units collected in additive solution C. Use irradiated blood D. Wash red cells prior to transfusion

Wash red cells prior to transfusion Nearly all RBC units are leukocyte reduced by the blood collection centers. Washing RBCs prior to transfusion will remove additional leftover leukocytes from leukodepletion as well as cytokines and other proteins that may be causing the patient's febrile transfusion reaction. Forty-one day old RBCs will have increased cytokines and be more likely to cause a febrile reaction. Irradiation inactivates leukocytes in the blood product and prevents transfusion-associated graft versus host disease.

A 65-year-old female with a history of ankylosing spondylitis managed by as needed over-the-counter pain medications presents with fever and confusion. She is found to have a urinary tract infection. She recovers after treatment with intravenous fluids and antibiotics. However, her kidney function did not recover to normal values. Outpatient renal biopsy reveals papillary necrosis of the renal tubules. What is the safest choice of pain management for this patient if needed? A. Naproxen B. Aspirin C. Tramadol D. Celecoxib

Tramadol This patient likely had an acute renal injury from a urinary tract infection, superimposed on underlying analgesic nephropathy. Drugs metabolized by the liver such as tramadol are likely to be a safer option for people at risk of kidney injury. The active metabolite of tramadol is O-dimethyl tramadol which is produced in the liver and excreted by the kidneys. The likely cause for impaired kidney function from long-term analgesia overuse is a hypotensive insult from inhibition of the prostaglandin pathway. Inhibition of the vasodilatory effect from the prostaglandin pathway is the most recognized and accepted mechanism of hypoperfusion-related medullary ischemia. This is generally accompanied by papillary damage in the form of necrosis in the vast majority of cases. Most non-steroidal anti-inflammatory drugs (NSAIDs) are metabolically active primarily and excreted by the kidneys. All NSAIDs are equally implicated including salicylates such as aspirin and naproxen, along with COX-2 inhibitors such as celecoxib. The key to improving morbidity and mortality associated with analgesic nephropathy lies in its prevention.

A 40-year-old man presents with a two-week history of fever. His medical history is significant for metallic prosthetic valve placement six years ago. Vital signs are heart rate 100/min, respiratory rate 20/min, blood pressure 100/70 mmHg, and temperature 101 F (38.3 C). Physical examination demonstrates a new murmur in the mitral area. Investigations show a leukocyte count of 11,000/microL platelet count of 120000/microL, and an elevated CRP of 190 mg/dL. Multiple blood cultures yield methicillin-resistant Staphylococcus aureus (MRSA). He is started on intravenous vancomycin and gentamicin. On the second day, he develops a pruritic, confluent, erythematous rash over his chest, neck, and thighs immediately following the IV antibiotic infusion. His blood pressure is 90/60 mmHg, and his heart rate is 120 beats/min. IV medications are discontinued, and supportive management is initiated. Which of the following is the most likely pathophysiology for this patient's condition post-antibiotic infusion?A. T cell-mediated reactionB. Antibody-dependent cell-mediated cytotoxicityC. Immune complex deposition in the tissuesD. Mast cell and basophil degranulation

Two types of hypersensitivity reactions are associated with the use of vancomycin: vancomycin flushing syndrome (VFS) and anaphylaxis. Patients with infections treated with IV vancomycin are at risk of developing VFS. The clinical presentation of VFS can vary, ranging from minor pruritus to occasionally life-threatening symptoms. Symptoms may occur as soon as 4 minutes after initiating the first dose until up to 7 days after dose completion.VFS is caused by the direct degranulation of mast cells and basophils, resulting in the release of histamine independent of preformed IgE or complement. Plasma tryptase levels are not elevated in VFS; therefore, this can be used to differentiate it from other immunological reactions.VFS is associated with rapid IV infusion; thus, slow IV administration will minimize adverse effects. Premedication with antihistamines can prevent the occurrence of VFS.Antibody-dependent cell-mediated cytotoxicity is responsible for type 2 hypersensitivity reactions. In serum-sickness-like reaction, immune complexes are deposited in the tissues. Antibiotics like cefaclor and trimethoprim-sulfamethoxazole can result in this type of reaction. T cell-mediated reactions are responsible for type 4 hypersensitivity reactions, for example, contact dermatitis.

A 40-year-old male presents to the clinic for evaluation of left heel pain. He has no significant past medical history. He is an avid runner and recently started training for the marathon again. His vital signs and physical exam are unremarkable except for mild posterior left heel pain to palpation. His erythrocyte sedimentation rate and C-reactive protein are normal. Which of the following diagnostic tests can be used to help confirm the suspected diagnosis? A. Serum urate and alkaline phosphate levels B. X-ray of the left foot C. Ultrasound of the Achilles tendon D. Biopsy of the Achilles tendon

Ultrasound of the Achilles tendon The patient likely has overuse Achilles tendinopathy. The history of repetitive overloading of the tendon and a recent increase in his physical activity level is suggestive of the diagnosis. Tenderness above the calcaneal insertion site is highly suggestive as well. Historically, tendon pain with the associated decreased function was described as tendinitis. Tendinitis, by definition, implies that tendon injury is accompanied by an inflammatory response. In reviewing available histopathology studies that compare healthy tendons to injured ones, it is evident that these injured tendons appear to be in a degenerative state with few or no inflammatory cells. Tendinosis more appropriately defines this process. Diagnosis is usually made based on presenting history and examination. Serum C-reactive protein (CRP) and erythrocyte sedimentation rate (ESR) levels are not very specific tests, but they can help determine if there is an inflammatory cause such as spondyloarthritis. Ultrasound machines are being used more frequently, given the dynamic nature of the study. Several interventions used to treat tendinosis are also done under sonographic guidance. Specificity and sensitivity vary between sonographic operators as well as when examining different tendons in the body. Common ultrasound findings in tendinosis include increased spacing of the hyperechoic fibrillar lines, reduced echogenicity, thickening of the tendon, and neovascularization (via color Doppler). Magnetic resonance imaging (MRI) is also a valuable tool that can be used when evaluating tendinosis.

A 65-year-old male presents to the emergency department with a complaint of fever and chills that has lasted for at least 24 hours. The patient has an extensive medical history and is currently taking seven chemotherapeutic drugs to treat acute myeloid leukemia (AML). He states that his last chemotherapeutic treatment was 14 hours ago and now shows a temperature of 103 ºF. On physical examination, the patient appears ill and has a chemotherapeutic port in the left chest wall. The absolute neutrophil count is 180/microliter (reference range: 3,000-5,800/microliter). The emergency room physician orders a chest X-ray that is unrevealing and blood cultures that have been sent to the lab. What is the most appropriate next step for the management of this patient? A. CT of the abdomen and pelvis B. Vancomycin C. Linezolid D. Vancomycin and cefepime

Vancomycin and cefepime This patient is presenting with neutropenic fever. Due to having a port that is possibly infected, empiric coverage needs to be broad-spectrum and cover skin flora. Empiric therapy neutropenic fever must include an antibiotic that has gram-negative coverage as well as pseudomonal coverage. Broad-spectrum options with gram-negative coverage are meropenem, piperacillin-tazobactam, cefepime, and imipenem-cilastatin.

An 18-year-old boy is brought to the emergency department after being found unconscious in his hostel toom. He has been having the habit of using illicit drugs. His pulse rate is 67/minute and respiratory rate is 10/minute. Benzodiazepine overdose is suspected. When should this patient be treated with flumazenil? A. If the specific benzodiazepine and time of ingestion is known B. If the ingestion included another drug with central nervous system effects C. When there is little chance the patient has taken other drugs, and the patient is not using benzodiazepines chronically D. All patients with benzodiazepine overdose should receive flumazenil

When there is little chance the patient has taken other drugs, and the patient is not using benzodiazepines chronically Flumazenil can cause seizures in patients who have ingested other drugs or who have been taking benzodiazepines for a long time. Flumazenil works instantly and reverses the sedation caused by benzodiazepines. It is important to monitor the patient afterward as repeat doses may be required. The risk of seizures is overstated, but the patient should be monitored for seizures, especially if the dose of benzodiazepine was high.


Related study sets

Chapter 54: Concepts of Care for Patients With Problems of the Biliary System and Pancreas

View Set

CMS Lecture 1: Intro to Cost Accounting

View Set

MIE CH5: Small Business, Entrepreneurship, and Franchising

View Set

strength and conditioning exam 4

View Set

Unit 2 (adroit, amicable, averse...)

View Set

Unit 4: Chapter 13 (Endocrine System)

View Set

Newton's Universal Law of Gravitation

View Set

AP LANGUAGE AND COMPOSITION MIDTERM GLOSSARY

View Set

Biology 1001 chapter2 Thinking Critcally

View Set

Perioperative: Chapters 17, 18, 19

View Set